Preview (15 of 93 pages)

ATI Nursing Care of Children Proctored exam, ATI Nursing Care of Children
Practice A, RN Nursing Care of Children Practice 2019 A ATI, ATI RN
Nursing Care of Children Online Practice 2019 B
1. What is a dictorial or authoritarian parenting style?
Answer: Parents try to control the child's behaviours and attitudes through unquestioned rules
and expectations
2. What is an authoritative parenting style?
Answer: also known as democratic, parents direct the child's behavior by setting rules and
explaining the reason for each rule setting
3. What is passive parenting?
Answer: parents are uninvolved, indifferent, and emotionally removed
4. A nurse manager on a paediatric floor is preparing an education program on working with
families for a group of newly hired nurses. Which of the following should the nurse include
when discussing the developmental theory?
A. describes that stress is inevitable
B. emphasizes that change with one member affects the entire family
C. provides guidance to assist families adapting to stress
D. Defines consistencies in how families change
Answer: D
5. A nurse is assisting a group of parents of adolescents to develop skills that will improve
communication. The nurse heads one parent states "my son knows he better do what I say".
Which of the parenting styles is he exhibiting?
A. Authoritarian
B. Permissive
C. Authoritative

D. Passive
Answer: A. Authoritarian
6. A nurse is performing family assessment. Which of the following should the nurse include?
(select all that apply)
A. medical history
B. parents' education level
C. child's physical growth
D. Support systems
E. Stressors
Answer: A, B, D, E
7. What is the expected pulse rate of a newborn?
Answer: 80 to 180/min
8. What is the expected pulse of a baby 1 week to 3 months?
Answer: 12 to 180/min
9. What is the expected pulse of a child 3 months to 2 years?
Answer: 70 to 150/min
10. What is the expected pulse of a child 2 to 10 years?
Answer: 60 to 110/min
11. What is the expected pulse of a child 10 years and older?
Answer: 50 to 90/min
12. What are the expected respirations of a newborn to one year?
Answer: 30 to 35/min
13. What are the expected respirations of a 1 to 2 year old?

Answer: 25 to 30/min
14. What are the expected respirations of a 2 to 6 year old?
Answer: 21 to 25/min
15. What are the expected respirations of a child 6 to 12 years old?
Answer: 19 to 21/min
16. What are the expected respirations of a 12 year old and older?
Answer: 16 to 19/min
17. What are the normal vitals of an infant?
Answer: HR: 80-180
RR: 30-35
BP: 65-80/40-50
18. Fontanels
Answer: should be flat and soft, posterior closes between 6 and 8 weeks, anterior closes between
12 and 18 months
19. Teeth
Answer: 6 to 8 teeth by 1 year of age, 20 baby teeth and 32 permanent teeth
20. How long is the Moro reflex present?
Answer: until 4 months of age
21. How long is the Tonic neck reflex present?
Answer: until 3 to 4 months of age
22. How long does the Babinski reflex last?
Answer: usually until a year

23. Expected findings of the olfactory (I) nerve in infants , children, and adolescents
Answer: Infants: difficult to test Children and Adolescents: identifies smell through each nostril
individually
24. Expected findings of optic nerve (II)?
Answer: Infants: looks at face and tracks with eyes
Children and adolescents: has intact visual acuity, peripheral vision, and colour vision
25. Expected findings for trigeminal nerve?
Answer: infants: has rooting and sucking reflex
children and adolescents: is able to clench teeth together and can detect touch on face with eyes
closed
26. A nurse is preparing to assess a preschool-age child. Which of the following is an appropriate
action by the nurse to prepare the child?
A. Allow the child to role play using miniature equipment
B. use medical terminology to describe what will happen
C. separate the child from her parents during examination
D. keep medical equipment visible to the child
Answer: A
27. A nurse is checking the vital signs of a 3-year-old during a well child visit, which of the
following findings should the nurse report to the provider?
A. temperature 37.2C (99.0F)
B. Heart rate of 106/min
C. Respirations 30/min
D. Blood pressure 88/54 mmHg
Answer: C
28. A nurse is assessing a child's ears. Which of the following is an expected finding?

A. Light reflex is located at the 2 o clock position
B. Tympanic membrane is red in colour
C. bone landmarks are not visible
D. Cerumen is present bilaterally
Answer: D
29. A nurse is assessing a 6-month-old infant. Which of the following reflexes should the infant
exhibit?
A. Moro
B. Plantar grasp
C. Stepping
D. Tonic neck
Answer: B
30. A nurse is performing a neurological assessment on an adolescent. Which of the following is
an appropriate reaction by the adolescent when the nurse checks the trigeminal cranial nerve?
(select all that apply)
A. clenching the teeth together tightly
B. recognizing a sour taste
C. identifying smells through each nostril
D. detecting facial touches when eyes closed
E. Looking down and in with the eyes
Answer: A, D
31. What happens to a baby's birth weight?
Answer: it should double by 6 months and triple by 1 year
32. How do infants grow?
Answer: 1 inch per month (2.5cm) for 6 months, then by 12 months, height/length should be
doubled

33. When do the first teeth abrupt?
Answer: between 6 and 10 months
34. Gross and fine motor by 3 months
Answer: only have slight head lag
35. Gross and fine motor by 4 months
Answer: should be able to roll from back to side
36. Gross and fine motor by 5 months
Answer: should be able to roll from front to back
37. Gross and fine motor by 6 months
Answer: should be able to roll from back to front and hold a bottle
38. Gross and fine motor by 7 months
Answer: move object from hand to hand
39. Gross and fine motor by 8 months
Answer: sit unsupported
40. Gross and fine motor by 9 months
Answer: crude pincer grasp
41. Gross and fine motor by 10 months
Answer: prone to sitting position and grasp a ratty by the handle
42. Gross and fine motor by 11 months
Answer: puts objects into a container and have a neater pincer grasp
43. Gross and fine motor by 12 months

Answer: tries to build a 2 block tower and won't succeed
44. What Piaget cognitive development stage are infants in?
Answer: Sensorimotor stage, birth to 24 months, separation, object permanence around 9
months, mental representation
45. How many words should the infant know?
Answer: 3-5 words and has concept of numbers by 1 year
46. What is the Erikson's stage of development for infants?
Answer: Trust vs. Mistrust, birth to 1 year, caretake meeting the needs of the infant
47. When does separation anxiety begin to occur?
Answer: between 4 and 8 months
48. When is there stranger fear in infants?
Answer: 6 to 8 months
49. What toys should be used for an infant?
Answer: rattles, blocks, brightly coloured toys, mirrors, patty cake
50. Infant Immunizations
Answer: Birth: hep B
2 months: hep B, IPV, RV, PCV, dtap, HIB
4 months: all of the 2m, hep B
6 month: all the previous
Flu shots: 6m to 1 year
51. Infant Nutrition

Answer: breast milk first 6 months, solids 4 to 6 months and first solid is usually iron fortified
rice cereal, no juice or water is needed for first year, foods introduced one at a time over 4-7 day
period to monitor for allergies
52. What are infant safety concerns?
Answer: choking/aspiration (grapes, coins, candy) burns (sunscreen, handles turned away from
stove, electrical outlets are covered), drowning, rear facing care seat until 2 years, crib slats are
no more than 6cm, no pillows, and sleep on back
53. A nurse is assessing a 12 months old infant during a well-child visit. Which of the following
findings should the nurse report to the provider?
A. closed anterior fontanel
B. eruption of 6 teeth
C. Birth weight doubled
D. Birth length increased by 50 %
Answer: C
54. A nurse is performing a developmental screening of a 10-month old infant. Which of the
following fine motor skills should the nurse expect to find? (select all that apply)
A. grasp a raddle by the handle
B. try building a two-block tower
C. use a crude pincer grasp
D. Place objects into a container
E. Walkes with one hand held
Answer: A, C
55. A nurse is conducting a well-baby visit with a 4 month old infant. Which of the following
immunizations should the nurse plan to administer? (select all that apply)
A. MMR
B. IPV
C. PCV

D. varicella
E. RV
Answer: B, C, E
56. A nurse is providing education about introducing new foods to the parents of a 4 months old
infant. The nurse should recommend that the parents introduce which of the following foods
first?
A. Strained yellow vegetables
B. Iron fortified cereals
C. Pureed foods
D. Whole Milk
Answer: B
57. A nurse is providing teaching about dental care and teething to the parent of a 9-month-old.
Which of the following statements by the parent indicates an understanding of the teaching?
A. I can give my baby a warm teething ring to relieve discomfort
B. I should clean my baby's teeth which a cool, wet washcloth
C. I can give Advil for up to 5 days while my baby is teething
D. I should place diluted juice in the bottle my baby drinks while falling asleep
Answer: B
58. Weight gain of toddler
Answer: 4x their birth weight by 30 months
59. Height gain of toddler
Answer: 3 inches per year (7.5cm)
60. Head and chest growth of toddler
Answer: head and chest circumference are about equal, compared to when they are born, and
their head is wider than their chest

61. gross and fine motor skills of a 15-month toddler
Answer: expect walking without help, should be able to build a 2 block tower
62. gross and fine motor skills of 18-month toddler
Answer: can throw a ball over head
63. gross and fine motor skills of 2 year old toddler
Answer: can walk up and down stairs by placing feet on each step and build a 6-7 block tower
64. gross and fine motor skills of 2.5 year old toddler
Answer: can jump with both feet and draw circles
65. Language of a toddler
Answer: 1 year = 1 word sentences/hollow phrases
2 year = 2-3 word sentences
66. Erikson's stage of Toddlers
Answer: autonomy vs shame and doubt, independence, begin to express selves by saying no a
lot, thrive on rituals, maintain routines
67. Toddler appropriate activities
Answer: blocks, push pull, thick crayons, puzzle
68. Bathroom needs of toddlers
Answer: toilet training begins when they have recognized the sensation that they need to go
potty
69. Immunizations of a toddler
Answer: 12-15 month: IPV, PCV, MMR, varicella, HIB 12-23 months: Hep A 2 doses 6 months
apart 15-18months: dtap and annual flu

70. Nutrition of a toddler
Answer: breast milk or formula through 1 year, 1-2 year whole milk, after 2 can transition to low
fat, limit juice consumption to 4-6oz per day, prevent choking, nuts, grapes, hot dogs, peanut
butter, raw carrots, tough meat, popcorn
71. What are safety hazards of toddlers?
Answer: burns, drowning, falls, aspiration, prevention
72. A nurse is assessing a 2.5-year-old toddler at a well-child visit. Which of the following
findings should the nurse report to the provider?
A. height increased by 7.5cm or 3inches in the past year
B. Head circumference exceeds chest circumference
C. anterior and posterior fontanels are closed
D. current weight equals four times the birth weight
Answer: B
73. A nurse is performing a developmental screening on an 18 month old. Which of the following
skills should the toddler be able to perform? (select all that apply)
A. build a tower with 6 blocks
B. Throw a ball overhead
C. walk up and down stairs
D. draw circles
E. use a spoon without rotation
Answer: B, E
74. A nurse is providing teaching about age-appropriate activities to the parent of a 2 year old.
Which of the following statements by the parent indicates an understanding of the teaching?
A. I will send my child's favourite studded animal when she will be napping away from home
B. My child should be able to stand on one foot for a second
C. The soccer team my child will be playing on starts next week
D. I should expect my child to be able to draw circles

Answer: A
75. A nurse is providing anticipatory guidance to the parents of a toddler. Which of the following
should the nurse include? (select all that apply)
A. Develop food habits that will prevent dental caries
B. Metting caloric needs resulting in an increased appetite
C. expression of bedtime fears is common
D. Expect behaviours associated with negativism and ritualism
E. Annual screenings for phenylketonuria are important
Answer: A, C, D
76. Growth of preschoolers
Answer: 4.5-6.5 pounds per year
2.3-3.5 inches per year or 6-9cm
77. Gross motor skills of 3 year old preschooler
Answer: can ride tricycle and jump off bottom step on stairs
78. Gross motor skills of 4 year old preschooler
Answer: can skip and hop on one foot and throw the ball over head
79. Gross motor skills of 5 year old preschooler
Answer: can jump rope
80. Cognitive development of the preschooler?
Answer: Piaget, preoperational phase 4-7 years, moving from preconceptual phase to the phase
of intuitive thought, magical thinking, animism, centration, time
Erikson, initiative vs guilt
81. Regression

Answer: in preschoolers, another baby in the family can cause the preschooler to regress to bed
wetting or thumb sucking, to be expected
82. What are appropriate activities for a preschooler?
Answer: playing ball, puzzles, tricycles, dress up, role playing
83. Immunizations of preschooler
Answer: 4-6 years, dtap, mmr, IPV and annual flu
84. What does the sleep schedule of a preschooler look like?
Answer: 12 hours of sleep, bedtime routine
85. Teeth of preschooler
Answer: eruption of primary teeth is finalized by the beginning of the preschool years
86. What may be a safety initiative for preschooler?
Answer: protective gear with tricycles
87. A nurse is providing teaching to the parent of a preschool age s child about methods to
promote sleep. Which of the following statements by the parent indicated an understanding of
the teaching?
A. I will sleep in the bed with my child if she wakes up during the night
B. I will let my child stay up and additional 2 hours on weekend nights
C. I will get my child watch television for 30 minutes must before bedtime each night
D. I will keep a dim lamp on in my child's room during the night Answer: D
88. A nurse is conducting a well child visit with a 5 year old child. Which immunizations should
the nurse plan to administer to the child? (Select all that apply)
A. DTaP
B. IPV

C. MMR
D. PCV
E. Hib
Answer: A, B, C
89. A nurse is preparing an education program for a group of parents of preschool-age children
about promoting optimum nutrition. Which of the following information should the nurse
include in the teaching?
A. saturated fats should equal 20% of total daily caloric intake
B. Average calorie intake should be 1800 calories per day
C. daily intake of fruits and vegetables should total 2 servings
D. Healthy diets include a total of 8g of protein each day
Answer: B
90. A nurse is performing a developmental screening on a 3 year old child. Which of the
following skills should the nurse expect the child to perform?
A. ride a tricycle
B. Hop on one foot
C. jump rope
D. throw a ball overhead
Answer: A
91. A nurse is caring for a preschool age child who says she needs to leave the hospital because
her doll is scared to be at home alone. Which of the following characteristics of preoperational
thought is the child exhibiting?
A. Egocentrism
B. Centration
C. Animism
D. Magical thinking
Answer: C

92. Growth of school age children
Answer: 4-6 pound weight gain per year and grown 2 inches (5cm) per year, permanent teeth
start to come in
93. Piaget cognitive development of school age children
Answer: concrete operations, perceptual to conceptual thinking, learns to tell time, see other
perspectives, solve problems
94. Erikson's stage of school age children
Answer: industry vs inferiority, trying to make meaning contributions to society and cooperative
and compete with others, peer groups important, competitive and cooperative play
95. What types of activities are appropriate for school age children?
Answer: board games, hop scotch, bikes, jump rope, organized sports
96. Immunizations of school age children
Answer: 11-12 years: DTaP, HPV vaccine (series of 3 shots)
97. How much sleep is recommended for a 12 year old?
Answer: 9 hours of sleep
98. What is a safety measure for school age children?
Answer: helmets
99. A nurse is discussing prepubescence and preadolescent with a group of parents of school-age
children. Which of the following information should the nurse include in the discussion?
A. initial physiologic changes appear during early childhood
B. changes in heigh and weight occur slowly during this period
C. growth differences between boys and girls become evident
D. signs of sexual maturation become highly visible in boys
Answer: C

100. A nurse is conducting a well child visit with a child who is scheduled to receive the
recommended immunizations for 11 to 12 year olds. Which of the following immunizations
should the nurse administer? (select all that apply)
A. TIV
B. PCV
C. MCV4
D. Tdap
E. RV
Answer: A,C, D
101. A nurse is teaching a course about safety during the school age years to a group of parents.
Which of the following information should the nurse include in the course? (select all that apply)
A. gating stairs at the top and bottom
B. wearing helmets when riding bicycles or skateboarding
C. riding safely in bed of pickup trucks
D. implementing firearm safety
E. wearing seat belts
Answer: B, D, E
102. Growth in adolescents
Answer: girls stop growing 2-2.5 years after their period starts, boys stop growing about 18-20
103. Sexual maturation is adolescent girls
Answer: breast development, pubic hair growth, underarm hair, period
104. Sexual maturation in adolescent boys
Answer: testicular enlargement, pubic hair, penile enlargement, underarm hair growth, facial
hair, vocal changes
105. Piaget cognitive stage of development for adolescents

Answer: formal operations
106. Erikson's stage of development for adolescents
Answer: identity vs role confusion
107. What are appropriate activities for adolescents
Answer: video games, music, sports, pets, reading
108. Immunizations for adolescents
Answer: flu, 16-18 years, meningitis before college
109. Injury prevention for adolescents
Answer: helmet use, seat belts, driving, substance abuse
110. Safe medication administration for children
Answer: oral is preferred, smallest measuring device possible, don’t mix oral meds in formula,
put in side of mouth, hold cheeks, and stroke chin to swallow
111. Administration of ear drops
Answer: pinna down and back
112. IM injections
Answer: preferred route is vastus lateralis, then the ventral gluteal or in the deltoid, 22-25 gauge
with half inch to 1 inch needle
113. IV safe administration
Answer: procedure room, away from bed, EMLA cream to numb area is recommended, avoid
terms like bee stink or stick, keep stuff out of sit, parents can stay, swaddle the infant, nonnutritive sucking is offered before, during, and after to infants

114. A nurse is providing teaching about expected changes during puberty to a group of parents
of early adolescent girls. Which of the following statements by one of the parents indicates and
understanding of the teaching?
A. girls usually stop growing about 2 years after menarche
B. girls are expected to gain about 65 pounds during puberty
C. girls experience menstartion prior to breast development
D. Girls typically grow more than 10 inches during puberty Answer: A
115. A nurse is providing anticipatory guidance to the parent of a 13 year old adolescent. Which
of the following screenings should the nurse recommend for the adolescent? (select all that
apply)
A. body mass index
B. blood lead level
C. 24 hour dietary recall
D. Weight
E. Scoliosis
Answer: A, D, E
116. A nurse is caring for an adolescent whose mother expresses concerns about her child
sleeping such long hours. Which of the following conditions should the nurse inform the mother
as requiring additional sleep during adolescents?
A. sleep terrors
B. rapid growth
C. elevated zinc levels
D. slowed metabolism
Answer: B
117. A nurse is teaching class about puberty in boys. Which of the following should the nurse
include as the first manifestation of sexual maturation?
A. pubic hair growth

B. vocal changes
C. testicular enlargement
D. facial hair growth
Answer: C
118. A nurse is planning to administer the influenza vaccine to a toddler. Which of the following
actions should the nurse take?
A. administer sub to the abdomen
B. use a 20guage needle
C. Divide the medication into two injections
D. place the child in supine position
Answer: D
119. A nurse is preparing to administer an IM injection to a child. Which of the following muscle
groups is contraindicated?
A. Deltoid
B. ventrogluteal
C. vastus lateral’s
D. dorsolateral
Answer: D
120. A nurse is teaching a parent of an infant about administration of oral medications. Which of
the following should the nurse include in the teaching? (select all that apply)
A. use a universal dropper for medication administration
B. as the pharmacy to add flavourings the medication
C. add the medication to a formula bottle before feeding
D. use the nipple of a bottle to administer the medication
E. hold the infant in a semi reclining position
Answer: B, D, E

121. A nurse is preparing to administer medication to a toddler. Which of the following actions
should the nurse take? (select all that apply)
A. identify the toddler by asking the parent
B. tell the parent to administer the medication
C. calculate the safe dosage
D. ask the toddler what toy he wants to hold during administration
E. offer juice after the medication
Answer: C, D, E
122. A nurse is caring for an infant who needs otic medication. Which of the following is an
appropriate action for the nurse to take?
A. Hold the infant in an upright position
B. pull the pinna downward and straight back
C. hyperextend the infants neck
D. ensure that the medication is cool
Answer: B
123. Paediatric pain management
Answer: self-report is only used for children 4 and older, FLACC scale is 2m to 7 years, pain
rate on a scale of 0-10 assessing behaviours of the child
FACES: 3 years and older
Oucher scale: 3-13
Numeric scale: 5 and older
use play therapy to explain procedures
Administer pain medications to kids routinely versus prn
combining opioid and non-opioid medications
EMLA cream, apply 1 hour prior to small stick or 2.5 hours before a big stick, occlusive dressing
over it
124. A nurse is competing a pian assessment of an infant. Which of the following pian scales
should the nurse use?

A. FACES
B. FLACC
C. Oucher
D. Non-communicating children’s pain checklist
Answer: B
125. A nurse is planning care for a child following a surgical procedure. Which of the following
interventions should the nurse include in the plan of care?
A. administer NSAIDS for a pain grater than 7 on a scale for 0 to 10
B. administer intranasal analgesics PRN
C. Administer IM analgesics for pain
D. administer IV analgesics on a schedule
Answer: D
126. A nurse is assessing an infant. Which of the following are manifestations of pain in an
infant? (select all that apply)
A. Pursed lips
B. loud cry
C. lowered eyebrow’s
D. Rigid body E. pushes away stimulus
Answer: B,C,D
127. A nurse is planning care for an infant who is experiencing pain. Which of the following
interventions should the nurse include in the plan of care? (select all that apply)
A. offer a pacifier
B. use of guided imagery
C. use swaddling
D. initiate a behavioral contract
E. encourage kangaroo care
Answer: A,C,E

128. A nurse is preparing a toddler for an IV catheter insertion using atraumatic care. Which of
the following actions should the nurse take? (select all that apply)
A. explain the procedure using the child's favourite toy
B. ask the parents to leave during the procedure
C. perform the procedure with the child in his bed
D. allow the child to make on choice regarding the procedure
E. apply lidocaine and prilocaine cream to three potential insertion sites
Answer: A, D, E
129. Hospitalization, illness, and play for the infant
Answer: stranger anxiety 6 to 18 months
130. Hospitalization, illness, and play for the toddler
Answer: behavior may regress, separation anxiety, intense reaction to procedure, parallel play
131. Hospitalization, illness, and play for the preschooler
Answer: magical thinking, they may think they caused an illness to happen, still experience
separation anxiety, explain the procedure in very simple clear language, give them a choice, if
possible (cup or spoon), associate play, paly together without much coordination
132. Hospitalization, illness, and play for the school age child
Answer: describe pain and increased ability to understand cause and effect, give factual info, tell
the truth, encourage contact with peer groups, and express feelings, cooperative play, play in
groups, more organized
133. Hospitalization, illness, and play in the adolescent
Answer: body image disturbance, feels isolated from peers, give factural info, tell the truth,
encourage contact with peer group, and express feelings, friends can come visit
134. A nurse is caring for a preschooler. Which of the following is the expected behavior of a
preschool-age child?

A. Describing manifestations of illness
B. relating fears to magical thinking
C. understanding cause of illness
D. awareness of body functioning
Answer: B
135. A nurse on a paediatric unit is caring for a toddler. Which of the following behaviours is an
effect of hospitalization? (select all that apply)
A. believes the experience is a punishment
B. experiences separation anxiety
C. displays intense emotions
D. exhibits regressive behaviours
E. Manifests disturbance in body image
Answer: B, C, D
136. A nurse is teaching a parent about parallel play in children. Which of the following should
the nurse include in the teaching?
A. children sit and observe others playing
B. Children exhibit organized play when in a group
C. the child plays alone
D. The child plays independently when in a group
Answer: D
137. A nurse is teaching a group of parents about separation anxiety. Which of the following
information should the nurse include in the teaching?
A. it is often observed int the school age child
B. detachment is the stage exhibited in the hospital
C. it results in prolonged issues of adaptability
D. kicking a stranger is an example
Answer: D

138. anticipatory grief
Answer: when death is expected or a possible outcome
139. complicated grief
Answer: extends for more than 1 year following the loss
140. parenteral grief
Answer: intense, long lasting, and complex
141. sibling grief
Answer: differs from adult/parenteral; depend on age and developmental stage
142. Infant/toddler view of death/dying
Answer: no concept of death
143. Preschoolers(3-6) view of death/dying
Answer: magical thinking, may feel guilt or shame, views dying as temporary
144. School age (6 to 12) view of death/dying
Answer: adult concept of death, express fear through uncooperative behavior
145. Adolescent (12-20) view of death and dying
Answer: adult concept of death, result of peers vs parents, stressed out by changes in physical
appearance
146. Physical manifestations of death
Answer: sensation of heat when body feels cool, decreased sensation, loss of senses, decrease
LOC, swallowing issues, bradycardia, hypotension, Cheyne stokes respirations
147. Nursing responsibility after death
Answer: allow family to stay with body, rock infant/toddler, assist in post-mortem care

148. A nurse is caring for child who is dying. Which of the following are findings of impending
death? ( select all that apply)
A. heightened sense of hearing
B. tachycardia
C. difficulty swallowing
D. sensation of being cold
E. Cheyne-stokes respirations
Answer: C, E
149. A nurse is teaching a parent about complicated grief. Which of the following statements
should the nurse make?
A. it is considered complicated gried if you are still grieving after 6 months
B. personal activities are affected when exercising complicated grief
C. parents will expire complicated grief together
D. complicated grief self-resolved in 12 months
Answer: B
150. A nurse is teaching a parent of a preschool child about factors that affect the child's
perception of death. Which of the following factors should the nurse include in the teaching?
A. Preschool children have no concept of death
B. Preschool children perceive death as temporary
C. preschool children often regress to an earlier stage of behavior
D. preschool children experience fear related to the disease process
Answer: B
151. A nurse often care for children who are dying. Which of the following are appropriate
actions of rhte nurse to take to maintain professional effectiveness? (select all that apply)
A. remain in contact with the family after their loss
B. develop a professional support system
C. take time off from work

D. suggest that a hospital representative attend the funeral
E. demonstrate feelings of sympathy toward the family
Answer: B, C
152. A nurse is caring for a child who has a terminal illness and revies palliative care with an
assistive personal (AP). Which of the following statements by the AP indicates understanding of
this review?
A. I’m sure the family is hopeful that the new medication will stop the illness
B. I’ll miss working with this client now that only nurses will be caring for him
C. I will get all the clients personal object out of his room
D. I will listen and response as the family talks about their child's life
Answer: D
153. Meningitis
Answer: Viral often resolves with supportive care, bacterial is more dangerous, PCV and Hib
vaccine help prevent, s/s include photophobia, n/v, irritability, h/a newborns: poor muscle tone,
weak cry, refusal to eat, vomiting, diarrhoea, poor sucking, possible fever or hypothermia, neck
is supple without nuchal rigidity, budging fontanels are a late sign
3m to 2years: seizures with a high pitch cry, fever and irritability, building fontanels, nuchal
rigidity, poor feeding and vomiting
2 years to adolescents: seizures, nuchal rigidity, fever and child, headache, n/v, irritability,
petechia, positive brudsinksi's sign (pull head forward - extremities will also flex (bro why are
you pulling on my neck)
positive kernigs signs - leg is flexed and you try to extend, it causes pain
154. Meningitis labs
Answer: CSF analysis through lumbar puncture, empty bladder before, EMLA cream, side lying
position, after procedure remain in bed 4-8 hours in a supine position
Bacterial: cloudy, increased WBC, increased protein, decrease glucose and +gram stain
Viral: clear, slightly elevated WBC, normal or slightly elevated protein, normal glucose, negative
gram stain

155. Nursing management for meningitis
Answer: It is suspected put them into droplet precautions, decrease LOC = NPO, provide quite
environment, dim the lights, seizure precautions, bacterial will need IV abx, maintain contact
precautions for bacterial for 24 hours after abx treatment has started, monitor for increased ICP
infants: bulging fintanels, increase in head circumference, high pitch cyr, bradycardia, adn
respirated changes
children: irritability, headache, n/v, seizures, bradycardia, and respiratory changes
156. Reyes syndrome
Answer: liver dysfunction and cerebral edema, associated with giving children aspirin for a
fever, follows a viral illness like the flu, gastroenteritis, or varicella, lab tests include: liver
enzymes (AST/ALT increase), increased ammonia levels, liver biopsy for diagnosis, CSF
analysis s/s: lethargy, irritability, confusion, deliriu, vomiting, LOC
157. A nurse is caring for a client who has suspected meningitis and a decreased level of
consiousness. Which of the following actions by the nurse is appropriate?
A. place the patient on NPO status
B. prepare the client for a liver biopsy
C. position the patient in dorsal recumbent
D. put the client in a protective environment
Answer: A
158. A nurse is reviewing cerebrospinal fluid analysis for a client who has suspected meningitis.
Which of the following findings should the nurse identify as indicating viral meningitis? (select
all that apply)
A. negative gram stain)
B. normal glucose content
C. Cloudy colour
D. decreased WBC count
E. normal protein count

Answer: A, B, E
159. A nurse is caring for a 4 month old infant who has meningitis, Which of the following
findings is associated with this diagnosis?
A. depressed anterior fontanel
B. constipation
C. presence of rooting reflex
D. high pitched crying
Answer: D
160. A nurse is caring for a patient who possibly has Reye syndrome. Which of the following is a
risk factor for developing reye syndrome?
A. recent history of infectious cystitis caused by candida
B. recent historical of bacterial otitis media
C. recent episode of gastroenteritis
D. Recent episode of Haemophilus influenzae meningitis
Answer: C
161. A nurse is developing an Inservice about viral and bacterial meningitis. The nurse should
include that the introduction of which of the following immunizations decreased the incidence of
bacterial meningitis in children? (select all that apply)
A. IPV
B. PCV
C. Dtap
D. Hib
E TIB
Answer: B, D
162. Risk factors for seizures
Answer: cerebral edema, fever, trauma, bleeding, toxins in body (lead), hypoglycaemia,
electrolyte imbalance, infection

163. What are the three phases of tonic-clonic seizures
Answer: tonic- arms and legs flex up and head and neck extend; stiff, LOC clonic- jerking
movements postical- awake and confused
164. absence seizure
Answer: school age children (4-12), loss of consciousness 5-10 seconds, daydreaming look,
drop what’s in hands, lip smacking or twitching of face
165. myoclonic
Answer: no postical
166. seizure diagnosis
Answer: EEG to find cause, prior to EEG no caffien and wash hair
167. seizure medications
Answer: carbamasepine, valporic acid, phenytoin, and diazepam
168. Complications of seizures
Answer: status epilepticus which a seizure lasting longer than 30 minutes, medical emergency
169. A nurse is caring for a child who has absence seizures. Which of the following findings
should the nurse expect? (select all that apply)
A. loss of consciousness
B. appearance of daydreaming
C. dropping held objects
D. falling to the floor E having a piercing cry
Answer: A, B, C
170. A nurse is caring for a child who just experienced a generalized seizure. Which of the
following is the priority action for the nurse to take?

A. Maintain the child in a side ying position
B. loosen the child’s restrictive clothing
C. reorient the child to the environment
D. not the time and characteristics of the seizure
Answer: A
171. A nurse is providing teaching to the parent of a child who is to have an EEG. Which of the
following responses should the nurse include in the teaching?
A. decaffeinated beverages should be offered on the morning of the procedure
B. do not wash your child's care the night before the procedure
C. with old all foods the morning of the procedure
D. give your child an analgesic the night before the procedure Answer: A
172. A nurse is teaching a group of parents about the risk factors for seizures. Which of the
following factors should the nurse include in the teaching? (select all that apply)
A. febrole episodes
B. hypoglycemia
C. sodium imbalances
D. low serum lead levels
E. presence of diphtheria
Answer: A, B, C
173. A nurse is reviewing treatment options with the parent of a child who has worsening
seizures. Which of the following treatment options should the nurse include in the discussion?
(select all that apply)
A. vagal nerve stimulator
B. additional antiepeltic medications
C. corpus callosotomy
D. focal resection
E. radiation therapy

Answer: A, B, C, D
174. minor head injury
Answer: confusion, vomiting, pallor, irritability or drowsiness, irritability is usually one of the
first signs of increased ICP
175. infant signs of head injury
Answer: budging fontanels, high pitched cry, poor feeding, increased sleeping, restlessness,
setting sun sign, distended scalp veins
176. children signs of head injury
Answer: nausea, vomiting, headache, seizures, blurred vision
177. late signs of head injury
Answer: delayed or impaired pupillary responses, posturing, decreased response to painful
stimuli, Cheyne-stokes respirations, optic dis swelling, decreased in LOC
178. interventions for head injury
Answer: stabilize the spine first, then vital signs, GCS, keep HOB 30, maintain head in
midline/neutral position, minimize oral or endotracheal suctioning, avoid coughing or blowing
their nose, foley catheter, stool softeners, implement seizer precautions
179. medications for head injuries
Answer: corticosteroids, mannitol (diuretic), anti-epileptics
180. surgical intervention for head injury
Answer: craniotomy, complications include haemorrhage, brain herniation, s/s of hernia include
loss of blinking, loss of gag reflex, unreactive pupils, coma, or resp arrect

181. A nurse is in the emergency department assessing a child following a motor vehicle crash.
The child is unresponsive, has spontaneous respirations of 22/min, and has a laceration of the
forehead that is bleeding. Which of the following nursing actions should be first?
A. Stabilize the neck first
B. cleanse the child's laceration with soap and water
C. implement seizure precautions for the child
D. initiate Iv access for the child
Answer: A
182. A nurse is caring for an adolescent who has a closed head injury. Which of the following
findings are indications of increased ICP? (select all that apply)
A. report of headache
B. alteration in pupillary response
C. increase motor response
D. increased sleeping E. increased sensory response
Answer: A, B, D
183. A nurse is caring for a child with ICP. Which of the following actions should the nurse take?
(select all that apply)
A. suction the endotracheal tube every 2hours
B. maintain a quiet environment
C. use two pillows to elevate the head
D. administer a stool softener
E. maintain body alignment
Answer: B, D, E
184. A nurse is assessing a child who has a concussion. Which of the following findings should
the nurse expect. (select all that apply)
A. amnesia
B. systemic hypertension
C. bradycardia

D. respiratory depression
E. confusion
Answer: A, C, D, E
185. A nurse is caring for a child who is taking mannitol for cerebral edema. Which of the
following adverse effects should the nurse monitor the child for and report to the provider?
A. bradycardia
B. weight loss
C. confusion
E. constipation
Answer: C
186. Snelling test
Answer: stand 10 feet away
187. myopia
Answer: near sightedness
188. hyperopia
Answer: far sightedness
189. strabismus
Answer: inward or outward deviation of one of the eyes, treatment is to patch the good eye
190. acute otitis media
Answer: middle ear infection, common under 7, their tubes are shorter and more horizontal than
adults, give pain meds, abx, and if surgery is needed, tube is put in the middle ear, myringotomy,
and the placement of tympanoplasty tubes, tubes will fall out on their out in 2-6 months and tel
HCP when they come out, don’t get their ears wet
191. Desired SpO2

Answer: 95-100%
192. Metered dose inhalers
Answer: shake 5-6 times, spacers make them more effective, hold inhaler up, while pressing in,
take a slow deep breath, and hold breath for 10 seconds before exhaling.
193. dry powder inhaler
Answer: don't shake
194. Chest physiotherapy
Answer: schedule 1 hour before or 2 hours after a meal to prevent vomiting, give bronchodilator
or nep treatment prior
195. Hypoxemia
Answer: s/s include tachypnoea, tachycardia, restlessness, accessory muscles, nasal flaring
196. O2 toxicity
Answer: leads to hypoventilation and maybe LOC
197. suctioning
Answer: clean technique for nasal and oral
198. ET tube and trach tube suctioning
Answer: high fowlers or fowlers, catheter one half of the diameter, hyperoxygenation and
hyperventilate with 100%, surgical aseptic, limit suctioning to 5 seconds for infants and 10
seconds for children, rest for 30to60 seconds in between passes
199. tonsilitis
Answer: fever meds, abx to cure the infection, culture to test for strep, tonsillectomy, side lying
position initially, assess for bleeding, frequent swallowing and clearing of the throat, clear fluids
after hag reflex has returned, no citrus juices, no milk products, discourage coughing or nose

blowing, warn parents that there may be some blood clots, limit strenuous. activity, full recovery
in about 2 weeks, grow beta of strep can leads to kidney infection or rheumatic fever
200. bacterial epiglottitis
Answer: drooling, hoarseness, difficulty speaking and swallowing, and high fever, most
important, do not put anything in their throat, no throat culture, or tongue balses, cause airway to
lcose up, abx therapy, intubation supplies ready
201. influenza
Answer: fever, body aches, congestion, antiviral (usually within first 48 hours)
202. complications of acute and infectious respiratory illnesses
Answer: pneumothorax and pleural effusion
203. bronchodilators for asthma
Answer: albuterol -> s/e is tachycardia and temors
204. anticholinergic like ipotropirum for asthma
Answer: s/e can’t see, can’t see, cant pee, cant spit, and shit
205. steroid for asthma
Answer: prednisode, rinse mouth after steroid inhaler because they can get a fungal infection
206. peak flow meters
Answer: stand up, 0 out machine, lips around device, blow out hard, 3x, highest reading
207. complications of asthma
Answer: status asthmatics, not relived by medications, intubation
208. cystic fibrosis

Answer: caused by a genetic mutation, autosomal recessive, both parent shave to carry recessive
traits, increased thick tenacious mucous, pancreas, lings, liver, small intestines, and reproductive
organs -carrel chest, finger clubbing, large loose fatty fould smelling stools (steatorrhea), not
gain weight, delayed growth, failure to thrive, deficiency of fat-soluble vitamins (ADEK), sweat
and tears are salty
209. diagnosis of cycstic fibrosis
Answer: sweat chloride test and DNA testing
210. cystic fibrosis treatment
Answer: Iv abx, and o2 therapy, diet high in calories, and protein, pancreatic enzymes with their
meals to help with digestion, pancrelipase and vitamin supplements, albuterol, anticholinergics,
dornase afa (decreases the viscocity of the mucous), chronic managing, parents find support
groups
211. Congenital heart defects usually result in 2 things
Answer: hypoxemia and heart failure, s/s include tachypnea, dyspnea, tachycardia, peripheral
edema, cyanosis, exercise intolerance, and polycythemia (increase in rbcs)
212. Increasing pulmonary blood flow defects
Answer: 1. ventricular septal defect (VSD)
a. creates a harsh murmur that can be heard at the left sternal border
2. atrial septal defect (ASD)
a. loud hard murmur; split sound
3. patent ductus arteriosus (PDA)
a. creates bounding pull
213. Obstructive blood flow defects
Answer: 1. pulmonary stenosis a. systolic ejection murmur
2. aortic stenosis
3. cortication of the aorta

a. upper body: bounding pulses and high pressure, flushed warm skin
b. lower body: low pressure, faint pulses, cool skin
214. decreasing pulmonary blood flow defects
Answer: 1. tricuspid atresia a. complete closure of the tricuspid valve also have to have an ASD
2. tet of fallot PROV a. pulmonary stenosis, VSD, overriding aorta, and right ventricular
hypertrophy
215. Mixed blood flow defects
Answer: 1. transposition of the great arteries a. sx within first 2 weeks of life - major cyanosis
2. truncus arteriosus a. no spetum between the ventricles b. requires sx after birth
3. hypoplastic left heart syndrome
216. EKG, echo, cardiac catheterization
Answer: Allergies to shellfish or iodine
• NPO 4-6 hours prior to procedure
• Both pedal pulses located
• Assess insertion site for bleeding
• lat position 4-8 hours post-op
217. Nursing care of cardiovascular disorders
Answer: • Frequent rest periods; cluster care
• Small frequent meals
• Crying kept to a minimum
• Encourage semi-fowlers or fowlers
• Car seat at 45-degree angle vs flat
• Feed Q3 hours
• Enlarged opening on bottle nipple
218. Medications for cardiovascular disorders

Answer: • Digoxin - help improve contractility of the heart Toxicity - n/v, halo. Decreased HR
and appetite
• ACE (Aprils) - help provide vasodilation
• Beta blockers - helps decrease HR, BP, and causes vasodilation
• Lasix - K levels High potassium foods: look in nutrition Severe hypoxemia episode knee to
chest to calm them down High risk for bacterial endocarditis - abx prior to dental and surgical
procedures
219. Rheumatic fever
Answer: Inflammation of the heart, blood vessels, and joints Caused by a strep throat infection
untreated or partially treated
• 2-6 weeks following Lab tests - throat culture, serum ASO titer, EKG, diagnosis based on the
jones criteria (pt. needs to have 2 major criteria or 1 major and 2 minor)
• Major criteria: carditis, subcutaneous nodules (nontender), polyarthritis, rash (pink-non pruritic
on the trunk and the inner surfaces of the extremities), Chorea - involuntary muscle movements
• Minor: fever + pain in one joint
220. Kawasaki disease
Answer: Acute systemic vasculitis - inflammation of the blood vessels Acute phase
• Onset of high fever that is unresponsive to meds, with development of other cm's
• Irritability, red eyes without drainage, bright red chapped lips, strawberry tongue, red oral
mucosa, red palms and feet, joint pain, enlarged lymph nodes, etc, Subacute phase
• Resolution of fever and gradual subsiding of other cm's
• Peeling skin Convalescent phase
• No cm's seen except abnormal labs
• Resolution 6-8 weeks from onset Treatment - IV Igg (gamma globulin)
• + aspirin to Avoid live immunizations for 11 months after the onset of the disease
221. Epistaxis

Answer: nose bleeding Sit upright and lean forward, pinch the nose for 10 min until bleeding
stops, ice on the nose, or cotton or tissue in the nare After the bleeding stop-Vaseline in the nose
to help prevent re-bleeding + recommend parent uses cool mist humidifier
222. Iron deficiency anaemia
Answer: Poor diet, drink a lot of cow milk (low in iron) Lab work - RBC, hbg & hct low Diet in
iron, protein, and vitamin C Iron supplement 1 hour before or 2 hours after milk or antacids,
vitamin C, straw, IM injection use z track method, stool is expected to turn tarry green color if
dose is adequate, brush teeth after Prevent overdosing of iron - locked in cabinet
223. Sickle cell anaemia
Answer: Autosomal recessive genetic disorder HbS is produced Increased blood viscosity,
obstruction of blood flow, tissue hypoxia Painful af Crisis - exacerbation African American
highest risks Fam history, reports of pain, SOB, pallor, jaundice (destruction of RBC's)
Vasoclusive crisis 4-6 days painful ischemia in the tissue Treatment: fluids and pain control
Blood products Complications CVA and any reactions to the blood products
224. Hemophilia
Answer: Prolonged bleeding time due to lack of a specific factor that's needed to clot properly A
= lack of factor 8 B = lack of factor 9 Excessive bleeding, joint pain and stiffness, bruising Labs:
prolonged PTT, platelets and prothrombin will be normal No rectal temps, avoid skin punctures
when necessary, hold pressure 5 min, painful joints = elevate and apply ice to that area Replace
the factors that are missing Minimize the risk of bleeding o RICE Complications: joint deformity
225. Rotavirus
Answer: most common cause of diarrhoea in kids 4 seconds, tachycardia, extreme thirst, mm very dry, tented skin, no tearing,
sunken eyes, sunken anterior fontanel, oliguria or anuria
229. Cleft lip
Answer: repaired in 2-3m o Before sx: use a wide based nipple for feeding, encourage
breastfeeding, squeeze cheeks together to decrease gap
• After sx: back and upright, elbow restraints, ns water or diluted hydrogen peroxide to clean the
suture line, antibiotic ointment if prescribed
230. Cleft palate
Answer: repaired in 6-1m
• Before sx: upright for feeing, one-way valve bottle with specially cut nipple for feeding, burp
frequently
• After sx: prone position, IV fluids then clear liquid for first 24 hours, nothing in mouth that
could mess up the sutures, elbow restraints
231. Complications of cleft
Answer: ear infections and hearing loss - seen by specialists, Speech therapists
Dental problems - teeth may not erupt normally

232. GERD - gastrointestinal reflex disease
Answer: Usually self resolves by 1 year of age s/s with infants - spitting up, irritability,
excessive crying, blood in the vomit, arching of back , stiffening, resp. problems, FTT, apnea s/s
with children - heartburn, abd pain, difficulty swallowing, chronic cough, noncardiac chest pain
nursing care small frequent meals, thicken infant's formula with rice cereal, avoid foods that can
make it worse, head elevated for 30 at least 1 hour after eating o meds: PPI (omeprazole,
pantoprazole); H2-receptor antagonists (famotidine or ranitidine) Sx - nissen fundoplication
233. Pyloric stenosis
Answer: thickening of the pyloric sphincter = obstruction Projectile vomiting, dehydration and
constant hunger, olive shaped mass in the RUQ Sx – pylorotomy
234. Hirschsprung's Disease
Answer: congenital aganglionic megacolon structural anomaly of the GI tract caused by lack of
ganglion cells in the segments of the colon resulting in decreased motility and mechanical
obstruction Ribbon like stool, vomiting bile, abd distention Risk factor infant fails to pass
meconium in 24-48 hours of life Nursing interventions - high calorie, protein & low fiber diet o
Surgery remove bad portion of the colon & may need a colostomy for a while
235. Intussusception
Answer: proximal segment of the bowel telescopes into a more distal segment, resulting in
lymphatic and venous obstruction causing edema in the area with progression, ischemia and
increase mucous into the intestine will occur Common in infants and children 3m-6 years Red
currant jelly stool, sausage shaped abd mass CF at risk Air enema is therapeutic procedure
236. Appendicitis
Answer: Avg age is around 10 years
Abd pain in the RLQ, ↓ or absent bowel sounds, fever, WBC ↑and inflammatory markers
Diagnosis with CT Avoid heat to the abd Fluids, abx prior to the stomach Suddenly feeling better
– ruptured appendix medical emergency

237. Enuresis
Answer: uncontrolled or unintentional urination after the age of 5 years for at least 3m's Primary
- never had control of bladder Secondary - have been potty trained and now they are wetting
their bed o Regression from stress of emotional trauma Self-esteem and coping strategies
Restrict fluids in the evenings, avoid constipation, etc.
238. UTI's
Answer: Frequent urination, foul smelling urine, fever, pallor, poor appetite, vomiting, increase
in thirst, swelling of the face, and seizures Diagnosis - urinalysis nitrates and leukocytes
elevation Educations - females wipe front to back, cotton underwear, avoid bubble baths, void
more, empty bladder fully Important constipation - high fiber diet
239. Bladder exstrophy
Answer: the bladder/urethra/ureteral orifices are coming through the suprapubic area - medical
emergency; requires immediate surgery Sterile gauze over that area and prepare for sx
240. Hypospadias
Answer: Urethral opening is on the underside (ventral) side of the penis No circumcision
241. Epispadias
Answer: Urethral opening on the upper part of the penis (dorsal) side No circumcision
242. Phimosis
Answer: narrowing of the opening of the foreskin can't retract the foreskin
243. Cryptorchidism
Answer: undescended testes Sx at 6 and 24m's
244. Hydrocele
Answer: fluid in the scrotal sac

245. Testicular torsion
Answer: medical emergency Enlargement of the effected testical and severe and sudden onset of
pain
246. Acute glomerulonephritis [AGN]
Answer: Associated with a strep infection Cloudy tea colored urine, dec urine output, periorbital
edema, facial edema that's worse in the am and then spreads down over the day, mild -severe
HTN, oliguria Proteinuria and Smokey or tea colored urine, hematuria, increased specific
gravity, ASO titer - for strep infection Nursing care
• Restrict sodium and fluid
• Edema risk for skin breakdown
• Diuretics, anti-hypertensives, abx for strep infection
247. Nephrotic syndrome
Answer: alteration in the glomerular membrane that allows proteins [esp. albumin] to pass
through to the urine resulting in decreased serum osmotic pressure Facial and periorbital edema,
dec urine, frothy urine, norm BP, >2+ protein, hypoalbuminemia, hyperlipidemia,
hemoconcentrion, hyponatremia maybe Daily weights - same scale, same time, same amount of
clothing on everyday Monitor edema - measure abd girth @ the level of the umbilicus Restrict
fluids and salt Skin breakdown Meds - steroids monitor for GI bleeding, hyperglycemia, etc o
Albumin and diuretics help increase the plasma volume and decrease edema in the pt.
248. Fractures
Answer: Open or compound - bone is sticking out of the skin Closed or simple - bone not
sticking out Complicated - organ or tissue is also damaged ABC's + elevated the extremity, apply
ice, stabilize the injured area & a complete neurovascular check
• Sensation
• Skin temp.
• Skin color
• Cap refill
• Pulses

• Movement Casting - elevate the cast above the level of the heart for the first 24-48 hours, apply
ice for 24 hours to dec swelling & turn and position the pt. every 2 hours to help dry the cast,
assess for inc warmth or hot spots on the cast - hot spot indicated infection Plaster casts use the
palms of your hands to avoid denting Expose all surfaces to promote drying Don't put anything
in the cast to itch
249. Traction care
Answer: Align, mobilize, and reduce muscle spasms in patients who have fractures Maintain
body alignment, give meds to help prevent muscle spasms and pain, neurovascular checks, pin
sites for s/s of infection, make sure the weights hang freely & not on the bed or floor, do not lift
or remove weights unless ordered Halo tractions - wrench attached to the vest if needed for CPR
250. Compartment syndrome
Answer: compression of the nerves, blood vessels, and muscle within a confined space Tissue
necrosis can occur Very intense pain unrelieved with meds, numbness, pulselessness, inability to
move digits, pallor, cool extremities Fasciotomy muscle compartment is cut open to allow tissue
to swell, decrease pressure, and restore blood flow 5 P's - pain, paresthesia, pulselessness,
paralysis, and paleness
251. Osteomyelitis
Answer: open/compound fractures - infection of the bone Fever, pain, tachycardia, edema Bone
biopsy Abx therapy to treat
252. Clubfoot
Answer: Treatment is serial casting
253. Legg-calve-perthes disease
Answer: aseptic necrosis of the femoral head (unit or bi) Intermittent painless limp, hip stiffness,
shortening of the effected leg, limited ROM Bracing, casting, or traction or replacement of the
hip joint

254. Developmental dysplasia of the hip [DDH]
Answer: Infants - asymmetry of the gluteal and thigh folds, limited hip abduction o + ortolani
test hip is reduced by abduction o + barlow test hip is dislocated by adduction Children - one leg
is shorted than the other, walk with a limp, + tendelenberg sign [while bearing weight on the
effected side, the pelvis tilts down], walks on tippy toes on one foot Newborn to 6 months =
Pavlik harness 12 weeks
• Check straps every 1-2 weeks for adjustments by HCP
• Preform neurovascular and skin checks
• Use an undershirt and wear knee socks
• Gently massage under the straps
• No lotion or powders
• Put diaper on under the straps Over 6 months
• Bryant harness Hips flexed at a 90-degree angle with the butt raised off the bed Maintain
traction & assure alignment Skin care
• Hip spica cast Neurovascular checks Position casts on the pillow & keep elevated until dry
Frequent position changes to promote even drying Handle casts with palm of hands to prevents
dents until dry Give sponge baths to avoid wetting the cast Use waterproof barrier around the
genital opening so nothing gets in there Complications from casts and harnesses - bowel and
bladder eliminations Fiber to help pooping and fluids
255. Osteogenesis imperfecta
Answer: an inherited condition that results in bone fractures and deformity along with restricted
growth Heterogeneous autosom dominant Brittle bone disease Multiple bone fracture, blue
sclera, early hearing loss, small discoloured teeth No cure - treatment is supportive Medication
pamidronate: can be used to increase bone density o s/e: hypocalcaemia, hypo magnesia, low
phosphate, low K, thrombocytopenia, dysrhythmias, kidney failure encourage the child to do low
impact exercises - braces and splints for support
256. scoliosis

Answer: lateral curvature of the spine and spinal rotation that causes rib asymmetry diagnosis bend at the waist with arms hanging while assessing for asymmetry of the rubs and flank
treatment - bracing or spinal fusion with rod placement
257. Cerebral palsy
Answer: impairment of motor function, coordination, and posture Abnormal perception and
sensation, visual, hearing, and speech impairments; seizures, and cognitive disabilities Cause is
unknown - correlated with prenatal risk factors Assessment findings:
• Spastic hypertonicity
• Dyskinetic (non-spastic, extrapyramidal) - jerking movements that appear slow and wormlike
of the trunk neck face and tongue
• Ataxic s/s - wide based gait and difficulty with coordination, difficulty with precise movements,
and low muscle tone Treatments - skeletal muscle relaxants [Baclofen] + valium [diazepam]
Complications - aspiration, elevated HOB, handle secretions, risk for injury
258. Spina bifida
Answer: failure of the osseous spine to close Neural tube defects are present at birth and effect
the CNS and osseous spine Occulta - not visible Cystica - protrusion of the sac is visible
Meningocele - contains spinal fluid and the meninges Myelomeningocele - contains spinal fluid,
meninges, and nerves Associated with a lack of folate acid during pregnancy s/s: o cystica protruding sac midline of the spine o occulta - dimpling of the lumbosacral area + port wine
angioma + dark hair tufts, subcutaneous lipoma interventions - close asap o sterile moist nonadherent dressing and change Q2 hours o prone position with hips flexed and legs abducted o no
pressure on the sac complications - skin ulceration, latex allergies , increased ICP, bladder issues,
and orthopedic issues
259. down syndrome
Answer: chromosomal abnormality small round head, flattened forehead, small nose with
depressed nasal bridge, small ears with short pinna, protruding abd, hypotonia and hyper
flexibility manage secretions and help prevent respiratory infections cardiac defects and

strabismus rinse mouth after feeding and throughout the day cool mist humidification and use
bulb syringe prn
260. juvenile idiopathic arthritis
Answer: chronic autoimmune inflammatory disease affecting the joints and other tissues joint
swelling, stiffness, redness and warmth worse in morning or after naps apply a splint for sleeping
encourage use of a firm mattress and discourage use of pillows apply heat or warm moist packs
to the affected joints encourage warm baths NSAIDS, methotrexate, steroids
261. Muscular dystrophy
Answer: group of inherited disorders with progressive degeneration of symmetric skeletal
muscle groups causing progressive muscle weakness and wasting Most common Duchenne's
MD - onset within 3-7years s/s: muscle weakness, unsteady gait, waddling, lordosis, and delayed
motor skills development o frequent falling, learning difficulties, progressive muscle atrophy
resp. and cardiac difficulties around age 20 corticosteroids complications: resp. compromise
progressive weakening of the resp. muscles
262. Impetigo
Answer: Caused by staph Reddish macule that becomes vascular and can erupt forming dry
crusty's + itchy Direct contact Abx ointment, burow's solution
263. Cellulitis
Answer: Firm swollen red area of the skin and subcutaneous tissue Fever Abx and warm moist
Compresses
264. Tinea
Answer: fungal, Round red scaley patches, itchy in warm and moist areas Head - selenium
sulphide shampoo Topical antifungal Treat infected pets as well
265. Lyme disease

Answer: Bit by a tick carrying borrelia burgdorferi Stage 1 - 3-31 days: flu like s/s bullseye rash
at the bite area Stage 2 - after 31 days more systemic issues - paralysis, swelling in joints,
weakness Stage 3 - deaf, encephalopathy, arthritis, weakness, numbness and tingling, and speech
issues
266. Scabies
Answer: Itchiness, rash, thin pencil mark lines, pimples on trunk, blisters on palms and soles
Apply 5% permethrin cream everywhere + family + wash everything in hot water
267. Pediculosis capitits (lice)
Answer: Small red bumps on the scalp, nits (white specks) on the hair shaft Shampoo containing
1% permethrin + remove nits with special comb + wash everything in hot water Can't wash? Bag
for 14 days Boil hair products for 1 hour in lice killing solution
268. Dermatitis
Answer: diaper rash Washing with warm water and mild soap Expose to air Encourage parents
to use good diapers + frequent changes No bubble baths Skin barrier - zinc oxide containing
Corn starch to reduce friction NOT TALCUM POWDER
269. Poison ivy
Answer: Plant exposure - treat area with alcohol followed by water then mild soap and water
Then apply a calamine lotion or a burrow solution Steroid gel
270. Seborrheic dermatitis
Answer: cradle cap Scaly and greasy thick flakes Not contagious Gently scrub the scalp with
mild shampoo or special treatment Fine tooth comb Keep nails trimmed short for skin issue kids
+ gloves or socks over hands for sleeping Cotton clothing Avoid excessive heat Avoid irritants
271. Atopic dermatitis - eczema
Answer: Intense itchy Damage from so much itching Antihistamines + topical steroids

272. Acne
Answer: Good diet, exercise, mild cleanser, don't pick Meds
273. Nursing interventions for burns:
Answer: ABC's IV access with lg bore catheters Immunization status - tetanus in last 5 years =
they are going to get one Advise family nothing greasy on burn
Fluids based on urine output Kids 30 kg 30ml/hour LR and
NS sometimes Manage pain - IV opioid's Nutrition - increase protein and calories + vitamin A,
C, and Zinc Restoring mobility - active and passive ROM Silver sulfadiazine - 2nd and 3rd
degree transient neutropenia Allograft - cavader Xenograft - animals Autograft - own skin
274. Hypoglycemia s/s: cold and clammy need some candy <60 blood sugar
Answer: Hunger Shakiness Diaphoresis Irritability Pale cool skin Possible in LOC Slurred
speech, HA, seizures Tachycardia and palpitations Normal to shallow respirations
275. Hyperglycemia s/s: hot and dry, sugar is high
Answer: Polyuria Polydipsia Polyphagia Dehydration Tired/fatigued Weak Nausea, vomiting,
abd pain Weak pulse and diminished reflexes Warm, dry, and flushed skin Rapid, deep
respiration - kussmauls respirations - fruity smelling breath
276. Diabetes mellitus diagnostic criteria
Answer: 8 hour fasting blood glucose of 126 or higher o No antidiabetic meds until after the
procedure A random blood glucose of 200 or more + classic s/s of diabetes Oral glucose
tolerance test of 200 or more in a 2-hour sample
• Balanced diet for 3 days prior
• Fast for 8 hours
• A fasting level is drawn at the start of the test
• Then instructed to consume a specific amount of glucose - and blood levels are drawn every 30
minutes for 2 hours
• Assess for hypoglycemia throughout the procedure HbA1c [glycosylated hemoglobin

• Expected range is 4-5.9% but an acceptable range for a child with diabetes can be 6-5-8% with
a goal of 240 or with a fever 102 & if ketones are in urine, rapid breathing, or confusion
279. Hypoglycemic patients [<60]
Answer: Treat with 10-15-gram simple carbohydrates [1 table spoon of sugar] o 4 oz. of orange
juice, 8 oz. of milk, 3-4 glucose tablets, 4 oz. regular soft drink Unconscious patients glucagon
IM or subq + give a simple carb once they wake up
280. Complications of diabetes mellitus
Answer: DKA life threatening condition when BG is over 330 & usually due to an acute illness,
non-compliance, or stress o Ketonemia + glycosuria + ketonuria + acidosis [pH 7.30 and bicarb
15] resulting in the breakdown of body fat for energy and an accumulation of ketones in the
blood, urine, and lungs o Rapid onset o Fruity breath, deep breathing, kussmauls, confusion,
dyspnea, n/v, dehydration, and electrolyte imbalances o Metabolic acidosis - hyperkalemia o
Treatment - as we are bringing the glucose levels down, the potassium levels may switch from
hyper to hypo Cardiac monitor Sodium bicarb for metabolic acidosis slow IV infusion When BG

levels get around 250 - add glucose to IV fluids in order to maintain 120-240 BG Give IV insulin
continuously Monitor levels hourly
281. growth hormone deficiency
Answer: short stature, delayed bone closure, and delayed sex development
282. growth hormone treatment
Answer: somatropin subcutaneous injections until the bones have closed
283. Immunizations
Answer: Common cold or minor illness - not contraindications for getting vaccinated Severe
acute illness - contraindicated Immunosuppression - contraindicated from a few Flu vaccine hypersensitivity to eggs = c/a Varicella - corticosteroids = c/a IPV - allergy to neomycin = c/a
MMR - allergy to gelatin & neomycin = c/a DTAP - occurrence of encephalopy, seizures, or
inconsolable crying that lasted a long time previously = c/a VL or ventral gluteal for smaller
children Older children - deltoid muscle Charting include date, route, site, type, manufacture lot
number, and expiration Low grade fever (common s/e) - don't give aspirin = Reyes syndrome
Babies vaccine can give concentrated oral sucrose solution on a pacifier 2 min before and for 3
min after the injection
284. A nurse is reinforcing home safety instructions with the parents of a toddler. Which of the
following parent statements indicates an understanding of the teaching?
Answer: "We will turn the pot handles toward the back of the stove."
The nurse should instruct the parents to turn pot handles toward the back of the stove to prevent
the toddler from pulling a pot off the stove, resulting in a burn.
285. A nurse is caring for an adolescent client who is a practicing Jehovah's Witness and is
scheduled for surgery for a ruptured appendix. The adolescent tells the nurse that based on her
religious beliefs, she cannot receive a blood transfusion. Which of the following responses
should the nurse make?
Answer: "Let's discuss the possibility of you needing a blood transfusion with your parents."

The nurse should offer to involve the child's parents to understand the family's beliefs about
blood transfusions.
286. A nurse is reviewing the laboratory report of a preschooler. Which of the following
laboratory results should the nurse report to the provider?
Answer: Lead 14 mcg/dL
This lead level is above the expected reference range for a preschooler. Therefore, the nurse
should report this result to the provider.
287. A nurse is collecting data from a 12-month-old infant during a well-child visit. The nurse
should identify which of the following findings as a deviation from expected growth and
development?
Answer: Birth weight doubled
The nurse should identify this finding as a deviation from expected growth and development.
The infant's birth weight should triple by 12 months of age. Therefore, the nurse should report
this finding to the provider.
288. A nurse is reinforcing discharge teaching with the guardian of a school-age child who has
acute lymphocytic leukemia and an absolute neutrophil count of 450/mm3. Which of the
following instructions should the nurse include?
Answer: "Keep your child away from crowded areas."
The nurse should instruct the guardian to keep the child away from crowds and visitors who have
an illness to decrease the risk for infection.
289. A nurse in a clinic is collecting data from an adolescent who has received all recommended
immunizations through the age of 6 years. Which of the following immunizations should the
nurse plan to administer?
Answer: Tetanus, diphtheria toxoids, and acellular pertussis (Tdap)
The Tdap vaccine is recommended between the ages of 11 and 12 years. Therefore, this
adolescent should receive the Tdap vaccine now.

290. A nurse is reinforcing teaching with the parent of a child who has a new prescription for
ferrous sulfate. The nurse should reinforce that the parent should administer the medication with
which of the following fluids to enhance the medication absorption?
Answer: Orange juice
The nurse should reinforce with the parent that administering ferrous sulfate with orange juice
will enhance medication absorption.
291. A nurse in a pediatric clinic is observing for an anaphylactic reaction after administering an
IM antibiotic to a child 5 min ago. Which of the following manifestations should the nurse
expect to observe first?
Answer: Hives
The nurse should observe for hives first because this is an early manifestation of an anaphylactic
reaction.
292. A nurse is reinforcing dietary teaching about a low-sodium diet with the parents of a child
who is recovering from acute glomerulonephritis. Which of the following food choices by the
parents indicates an understanding of the teaching?
Answer: Apples
The nurse should instruct the parents that apples are low in sodium and supply the child with
energy needed for recovery.
293. A nurse is collecting data from a 12-month-old infant during a well-child visit. Which of the
following findings should the nurse report to the provider?
Answer: BP 115/70 mm Hg
The nurse should identify that this blood pressure is above the expected reference range for a 12month-old infant and report this finding to the provider.
294. A nurse is reinforcing discharge teaching with the guardian of a child who has juvenile
idiopathic arthritis (JIA). Which of the following statements by the parent indicates an
understanding of the teaching?
Answer: "I will have my child sleep in knee, wrist, and hand splints."

The nurse should reinforce with the guardian that splinting the child's joints at night will
decrease pain and enhance joint function.
295. A nurse is collecting data about the dietary habits of an adolescent client. The nurse should
identify that which of the following findings puts the client at risk for nutritional deficits?
Answer: The client skips eating dinner for track practice three times per week.
The nurse should identify that adolescents are often at risk for developing poor eating habits.
Skipping dinner twice each week puts this client at risk for nutritional deficits.
296. A nurse is reinforcing teaching with the guardians of a school-age child who has hearing
loss. Which of the following techniques should the nurse recommend to facilitate communication
with the child?
Answer: Speak at the child's eye level. The nurse should instruct the guardian to speak at the
child's eye level and ensure that there is adequate lighting on the speaker's face to facilitate
lipreading and communication.
297. A nurse is assisting with a sterile dressing change for an adolescent who has a partial
thickness burn on the right hip. Which of the following actions should the nurse take first?
Answer: Administer pain medication to the client. According to evidence-based practice, the
nurse should first provide pain medication to the client to reduce discomfort during the
procedure.
298. A nurse is reviewing the laboratory report of a school-age child who is receiving
prednisone. Which of the following laboratory results should the nurse report to the provider?
Answer: Sodium 150 mEq/L
Hypernatremia is an adverse effect of prednisone. This level is above the expected reference
range for a school-age child. Therefore, the nurse should report this value to the provider.
299. A nurse in a community center is reinforcing teaching about poison control with a group of
parents. A parent asks what to do if a child ingests a large quantity of acetaminophen. Identify
the sequence of actions the nurse should recommend to the parent.

Answer: Determine if the child is breathing.
Empty the child's mouth of remaining pills and residue.
Identify the medication and dosage strength.
Call a poison control center.
The child's respiratory and cardiovascular status should be checked first to determine if CPR is
necessary. Then, the child's mouth should be emptied of pills and residue to prevent additional
exposure to the medication. Next, the parent should identify the medication and dosage strength
by looking at the medication container. Lastly, the parent should contact a poison control center
for advice on the next course of action.
300. A nurse is caring for a preschooler who has a new diagnosis of asthma. Which of the
following medications should the nurse instruct the parent to administer for an acute asthma
attack?
Answer: Albuterol
The nurse should inform the parent to administer albuterol, a short-acting beta2 agonist, to the
preschooler for acute asthma attacks.
301. A nurse is assisting with the care of an infant who has spina bifida and recently had a
ventriculoperitoneal shunt placed for hydrocephalus. Which of the following findings should the
nurse identify as an indication of increased ICP?
Answer: High-pitched cry
The nurse should identify that a high-pitched cry is an indicationof increased intracranial
pressure.
302. A nurse is reinforcing discharge teaching with the guardian of a school-age child who has a
new prescription for home oxygen therapy. Which of the following statements by the guardian
indicates an understanding of the teaching?
Answer: "I will make sure that electrical devices in the house are grounded."
This response by the guardian indicates an understanding of the nurse's instructions. Due to the
combustible nature of oxygen, all pieces of electrical equipment in the home should be grounded
to decrease the risk of a fire caused by an electrical spark.

303. A nurse is caring for a school-age girl who is being treated for frequent, severe UTI's. The
nurse should recognize that which of the following statements by the parent indicates a possible
cause of the UTI's?
Answer: "My daughter has bowel movements every 4 to 5 days."
The nurse should recognize that this frequency indicates the child is constipated. Therefore, large
stool masses might prevent complete emptying of the bladder and lead to urinary stasis and
infection.
304. A nurse is preparing to administer the measles, mumps, and rubella (MMR) vaccine to a
preschooler. The nurse should recognize which of the following statements by the parent as a
contraindication to receiving the immunization?
Answer: "My child received an immunoglobulin last month." The nurse should identify that a
preschooler who received an immunoglobulin less than 1 month ago should not receive the
MMR vaccine on this day. The nurse should instruct the parent to reschedule the immunization
after 3 months have elapsed, since the child received passive immunity via administration of an
immunoglobulin.
305. A nurse is reinforcing teaching with the guardian of a child who has scabies and a new
prescription for permethrin 5% cream. Which of the following information should the nurse
include?
Answer: "The medication will eliminate your child's itching within 2 to 3 weeks."
The nurse should instruct the guardian that, although the medication kills the mites, itching can
continue for 2 to 3 weeks following application of the medication.
306. A nurse is collecting data from an infant who has severe dehydration. Which of the
following findings should the nurse expect?
Answer: Weight loss of 10%
The nurse should expect an infant who has severe dehydration to experience weight loss of 10%
or greater.

307. A nurse is assisting with the administration of a nasogastric enteral feeding for an infant.
Which of the following actions should the nurse take?
Answer: Place the infant in semiFowler's position for 1 hr after the feeding.
The nurse should elevate the head of the infant's bed by 30º to 45º for 30 min to 1 hr after the
feeding.
308. A nurse is caring for a school-age child who has been admitted to the facility in sickle cell
crisis. The nurse is measuring the child's oral intake for the shift. The child consumed 4 oz of
juice at breakfast. For lunch, the child consumed 6 oz of milk, 6 oz of gelatin, and drank 7 oz of
water. What is the child's oral intake for this shift in mL?
Answer: 690 mL
1 oz = 30 mL
309. A nurse is assisting with the care of a 3-year-old child who is prescribed a lumbar puncture.
Which of the following actions should the nurse take to prevent complications?
Answer: Maintain the child in a flat position after the procedure. After a lumbar puncture, the
optimal position for the client is flat and supine to prevent headaches.
310. A nurse is collecting data from a 6-month-old child who is experiencing a sickle cell crisis.
Which of the following areas should the nurse observe when monitoring for manifestations of
splenic sequestration?
Answer: The nurse should observe the location over the infant's spleen (LUQ of abdomen) when
monitoring for manifestations of splenic sequestration. Splenic sequestration is an enlargement
of the spleen due to pooling of sickled cells in the blood.
311. A nurse is reinforcing teaching with an adolescent female client who has acne vulgaris and a
new prescription for isotretinoin. Which of the following information should the nurse include?
Answer: "You will need to have two negative pregnancy tests prior to starting this medication."
The nurse should reinforce with the client that isotretinoin is teratogenic. Pregnancy must be
ruled out prior to administration and before each subsequent refill. The client should use two
effective forms of contraception while taking this medication.

312. A nurse is assisting the provider with a developmental assessment of a toddler. Which of the
following behaviors should the nurse recognize as an expected finding?
Answer: Stands on one foot for several seconds Standing on one foot for several seconds is an
expected behavior for a toddler.
313. A nurse is reinforcing teaching about home safety with the parent of a toddler. Which of the
following parent statements indicates an understanding of the teaching?
Answer: "I will place a screen in front of the fireplace."
The nurse should instruct the parent to place a screen in front of a fireplace or other heating
appliances to prevent burns.
314. A nurse is reinforcing dietary teaching with the guardian of a school-age child who has
celiac disease. Which of the following foods should the nurse recommend including in the child's
diet?
Answer: White rice
The nurse should reinforce to the guardian that celiac disease is a genetic autoimmune disorder
in which eating gluten, even in very small amounts, can damage the child's small intestine.
Currently, the only treatment for the disease is a lifelong, strict adherence to a gluten-free diet.
The nurse should stress the importance of avoiding foods containing wheat, rye, barley, and oats.
The child should consume foods that are gluten-free, such as milk, cheese, rice, corn, eggs,
potatoes, fruits, vegetables, fresh poultry, meats, fish, and dried beans.
315. A nurse is collecting data from a toddler who has gastroesophageal reflex disease (GERD).
Which of the following findings should the nurse expect?
Answer: Chronic cough
The nurse should identify that a chronic cough is an expected finding in a child who has GERD.
316. A nurse is reinforcing teaching about injury prevention with the guardian of an infant.
Which of the following statements by the guardian indicates an understanding of the teaching?
Answer: "I should make sure my baby's clothing does not have buttons on it."

The nurse should instruct the guardian to avoid dressing the infant in clothing with buttons to
reduce the risk of choking and aspiration.
317. A nurse is reviewing the medical record of a female adolescent client who has primary
amenorrhea. Which of the following findings should the nurse identify as a risk factor for this
disorder? (Select all that apply.)
Answer: Hypothyroidism
The nurse should identify that hypothyroidism and other endocrine disorders are risk factors for
primary amenorrhea.
Cannabis use
The nurse should identify that cannabis use is a risk factor for primary amenorrhea. Oral
contraceptive use
The nurse should identify that oral contraceptive use affects the estrogen and progesterone cycle
and is a risk factor for primary amenorrhea.
Emotional stress
The nurse should identify that emotional stress causes hypothalamic suppression and is a risk
factor for primary amenorrhea.
318. A nurse is reinforcing teaching with the family of a preschooler whose parent has a terminal
diagnosis. Which of the following statements should the nurse include when discussing
ageappropriate responses to death?
Answer: "At this age, your child likely believes his thoughts can cause another person's death."
The nurse should reinforce that, at this age, the preschooler might believe that his thoughts can
cause another person's death, which can make him feel guilty or responsible for the death.
319. A nurse is reinforcing teaching regarding the immunization schedule of a newborn. Which
of the following statements made by the parent should the nurse recognize as an understanding
of the newborn's immunization schedule?
Answer: "My baby will receive his next immunization when he is 2 months old."
Newborns should receive the next scheduled immunization 2 months after birth.

320. A nurse is preparing to administer an enteral feeding to a child who has cerebral palsy and a
nasogastric tube. Which of the following actions should the nurse take?
Answer: Confirm that the pH of the stomach contents is 5 or less.
The nurse should test the pH of the stomach contents prior to administering the tube feeding in
order to confirm tube placement in the stomach. The nurse should identify that a pH of 5 or less
indicates gastric placement.
321. A nurse is assisting with scoliosis screenings for a group of school-age children. The nurse
should place the students in which of the following positions during the screening?
Answer: Bending forward with back parallel to the floor The nurse should observe for
asymmetry and prominence of the rib cage by having the students bend forward with the back
parallel to the floor.
322. A nurse is reviewing the laboratory report of a preschooler who has a Wilm's tumor and is
scheduled to begin treatment with an antineoplastic medication regimen. Which of the following
laboratory results should the nurse report to the provider?
Answer: Platelet count 70,000/mm3
This platelet count is below the expected reference range for a preschooler and increases the risk
for spontaneous bleeding. The nurse should hold the medication and report this finding to the
provider immediately.
323. A nurse in a pediatric clinic is collecting data from an infant who recently started taking
digoxin. Which of the following manifestations should the nurse identify as an indication of
digoxin toxicity and report to the provider?
Answer: Vomiting
The nurse should identify that vomiting, especially unrelated to feedings, is a manifestation of
digoxin toxicity and should be reported to the provider.
324. A nurse is reinforcing teaching with the parents of a 7-year-old female child about
behavioral expectations. Which of the following behaviors should the nurse include in the
teaching?

Answer: Spends a lot of time by herself
Spending time alone is an expected characteristic of a 7-year-old female child. When they do
spend time with others, children in this age group prefer to socialize with children of the same
sex and age.
325. A nurse is contributing to the plan of care for a child who has sickle cell anemia and is
experiencing a vaso-occlusive crisis. Which of the following is the priority intervention for the
nurse to recommend to include in the plan?
Answer: Promote oxygen utilization
The priority action the nurse should take when using the airway, breathing, circulation (ABC)
approach to client care is promoting oxygen utilization to prevent further sickling of the red
blood cells and promote adequate oxygenation of the tissue.
326. A nurse is contributing to the plan of care for an adolescent client who has human
immunodeficiency virus (HIV). Based on the adolescent's diagnosis, which of the following
actions should be included in the plan of care?
Answer: Inform the client regarding routes of transmission.
The nurse should inform the client about the transmission of HIV and how to prevent its spread.
327. A nurse is reinforcing teaching with the guardian of a child who has a new diagnosis of
enterobiasis. The nurse should advise the guardian to take which of the following actions to
prevent infection?
Answer: Trim the child's fingernails short.
The nurse should instruct the guardian to trim the child's fingernails short to reduce the collection
of eggs under her nails and prevent reinfection.
328. A nurse is collecting data about a 4-year-old preschooler's gross motor skills. The nurse
should expect the preschooler to be able to perform which of the following activities?
Answer: Hopping on one foot
The nurse should expect to find that a 4-year-old preschooler is able to hop on one foot.

329. A nurse is reinforcing discharge teaching with the parent of a school-age child who is being
treated for nephrotic syndrome. The parent asks the nurse why it is necessary to check the child's
urine for protein. Which of the following explanations should the nurse offer?
Answer: "A decrease in urine protein indicates that treatment is effective."
The desired outcome of steroid therapy in the treatment of nephrotic syndrome is a reduction of
proteinuria.
330. A nurse is reinforcing teaching about glucose monitoring with the parent of a child who has
type 1 diabetes mellitus. Which of the following instructions should the nurse include in the
teaching?
Answer: "Put your child's finger under warm, running water prior to collecting blood."
The nurse should instruct the parent that placing the child's finger under warm, running water
increases the blood flow to the finger, which will make it easier to obtain the sample.
331. A nurse in a pediatric clinic is collecting data from an infant who was recently exposed to
pertussis. The nurse should recognize which of the following as a manifestation of pertussis?
Answer: Dry cough
The nurse should identify that a dry cough is an early manifestation of pertussis.
332. A nurse is reinforcing teaching with the parent of a child who has hemophilia and is
experiencing acute hemarthrosis. Which of the following instructions should the nurse include in
the teaching?
Answer: Keep the affected joints immobilized
The nurse should reinforce with the parent to keep the child's affected joints elevated and
immobilized to minimize bleeding. After the acute episode, the child should begin active rangeof motion exercises.
333. A nurse is auscultating heart sounds on an infant. The nurse should identify this sound as
which of the following? (Audio clip)
Answer: Sinus rhythm

The nurse should identify this heart sound as sinus rhythm. The nurse should auscultate heart
sounds at the apical impulse, which is at the left midclavicular line and fifth intercostal space.
The expected heart sounds include S1, which is the closure of the atrioventricular valves, and S2,
which is the closure of the semilunar valves.
334. A nurse is preparing to administer ophthalmic drops to a child. Which of the following
actions should the nurse take?
Answer: Apply pressure to the lacrimal punctum for 1 min following administration.
The nurse should apply pressure to the lacrimal punctum to prevent the medication from entering
the nasopharynx.
335. A nurse has just received change-of-shift report for four children in a pediatric unit. Which
of the following children should the nurse collect data from first?
Answer: A child who has a fever and nuchal rigidity
A client who has a fever and nuchal rigidity is unstable. This finding indicates bacterial
meningitis, which requires urgent data collection and intervention to reduce complications for
the child and prevent further spread of the infection. Therefore, the nurse should collect data
from this child first.
336. A nurse is caring for a school-age child who has skeletal traction applied to the right lower
leg to repair a femur fracture. Which of the following findings is the priority for the nurse to
report to the provider?
Answer: Report of tingling in the right foot
The nurse should identify that the greatest risk to the child is nerve injury. Therefore, tingling in
the right foot, which can indicate nerve damage or compartment syndrome, is the priority finding
for the nurse to report to the provider.
337. A nurse is assisting with the care of a child who has tonic-clonic seizures. Which of the
following actions should the nurse take?
Answer: Have a suction canister and tubing available in the room.

The nurse should have a suction canister and tubing available in the child's room to keep the
child's airway patent during a seizure.
338. A nurse is monitoring a preschooler following an abdominal CT scan with contrast dye. The
nurse should identify which of the following as an indication that the preschooler experienced an
allergic reaction to the contrast dye?
Answer: Urticaria
The nurse should monitor the child for an allergic reaction to the contrast dye. Manifestations of
the allergic reaction include urticaria, itching, flushing of the skin, and possible anaphylaxis.
339. A nurse is collecting data from a child who has iron deficiency anemia. Which of the
following data signifies that adherence to ferrous sulfate therapy has occured?
Answer: Green, tarry stools
Green, tarry stools are an expected outcome of ferrous sulfate therapy. Therefore, this is an
indication of adherence to the prescribed medication regimen.
340. A nurse is reinforcing teaching with the parents of a child who has cystic fibrosis and is
taking pancrelipase as a pancreatic enzyme replacement. The nurse should plan to inform the
child's parents that the therapeutic effects of this medication can be evaluated by which of the
following?
Answer: Amount and consistency of stools
Recording the amount and consistency of the child's stools will help determine the effectiveness
of pancrelipase, which is taken to decrease the bulk of feces.
341. A nurse is reinforcing teaching with the parents of a toddler who has strabismus. Which of
the following treatments should the nurse plan to include in the teaching?
Answer: Eye patch
Treatment of strabismus includes covering the strong eye to strengthen the muscles in the weak
eye.

342. A nurse is reinforcing teaching about tracheostomy care with the parent of a toddler who has
a temporary tracheostomy. Which of the following instructions should the nurse include in the
teaching?
Answer: Ensure one finger fits between the ties and the neck.
The nurse should instruct the parent that one finger should fit between the ties and the neck to
ensure the tube is held securely in place.
343. A nurse is collecting data from a toddler at a well-child visit. Which of the following
findings should the nurse identify as a possible indication of child maltreatment?
Answer: Laceration on the side of the torso
A laceration on the side of the torso is not an injury that occurs due to the typical clumsiness of a
toddler. This finding indicates the need to further investigate for suspected child maltreatment.
344. A nurse is creating a plan of care for a school-age child who has heart disease and has
developed heart failure. Which of the following interventions should the nurse include in the
plan?
Answer: Provide small, frequent meals for the child.
The metabolic rate of a child who has heart failure is high because of poor cardiac function.
Therefore, the nurse should provide small, frequent meals for the child because it helps to
conserve energy.
345. A nurse is teaching the parent of an infant who has a Pavlik harness for the treatment of
developmental dysplasia of the hip. The nurse should identify that which of the following
statements by the parent indicates an understanding of the teaching?
Answer: "I will place my infant's diapers under the harness straps."
To prevent soiling of the harness, the parent should apply the infant's diaper under the straps.
346. A nurse is planning care for a school-age child who is in the oliguric phase of acute kidney
injury (AKI) and has a sodium level of 129 mEq/L. Which of the following interventions should
the nurse include in the plan?
Answer: Initiate seizure precautions for the child.

A sodium level of 129 mEq/L indicates hyponatremia and places the child at increased risk for
neurological deficits and seizure activity. The nurse should complete a neurologic assessment
and implement seizure precautions to maintain the child's safety.
347. A nurse is assessing a school-age child immediately following a perforated appendix repair.
Which of the following findings should the nurse expect?
Answer: Absence of peristalsis
The nurse should expect absence of peristalsis immediately following a perforated appendix
repair, until the bowel resumes functioning.
348. A nurse is preparing an adolescent for a lumbar puncture. Which of the following actions
should the nurse take?
Answer: Apply topical analgesic cream to the site 1 hr prior to the procedure.
The nurse should apply a topical analgesic to the lumbar site 1 hr prior to the procedure to
decrease the adolescent's pain while the lumbar needle is inserted.
349. A nurse is caring for a school-age child who is receiving cefazolin via intermittent IV bolus.
The child suddenly develops diffuse flushing of the skin and angioedema. After discontinuing
the medication infusion, which of the following medications should the nurse administer first?
Answer: Epinephrine
This child is most likely experiencing an anaphylactic reaction to the cefazolin. According to
evidence-based practice, the nurse should first administer epinephrine to treat the anaphylaxis.
Epinephrine is a beta adrenergic agonist that stimulates the heart, causes vasoconstriction of
blood vessels in the skin and mucous membranes, and triggers bronchodilation in the lungs.
350. A nurse is teaching the parent of a preschooler about ways to prevent acute asthma attacks.
Which of the following statements by the parent indicates an understanding of the teaching?
Answer: "I should keep my child indoors when I mow the yard."
The nurse should instruct the parent to keep the preschooler indoors during lawn maintenance or
when the pollen count is increased. Guarding against exposure to known allergens found

outdoors, such as grass, tree, and weed pollen, will decrease the frequency of the preschooler's
asthma attacks.
351. A nurse is proving dietary teaching to the parent of a school-age child who has celiac
disease. The nurse should recommend that the parent offer which of the following foods to the
child?
Answer: White rice
The nurse should recommend that the parent offer white rice to the child because it is a glutenfree food. The nurse should instruct the parent that the child will remain on a lifelong glutenfree
diet and the child should not consume oats, rye, barley, or wheat, and sometimes lactose
deficiency can be secondary to this disease.
352. A nurse is reviewing the laboratory report of a school-age child who is experiencing fatigue.
Which of the following findings should the nurse recognize as an indication of anemia?
Answer: Hematocrit 28%
The nurse should recognize that this hematocrit level is below the expected reference range of
32% to 44% for a school-age child. The child can exhibit fatigue, lightheadedness, tachycardia,
dyspnea, and pallor due to the decreased oxygen-carrying capacity.
353. A nurse is preparing to collect a sample from a toddler for a sickle-turbidity test. Which of
the following actions should the nurse plan to take?
Answer: Perform a finger stick.
The nurse should perform a finger stick on a toddler as a component of the sickle-turbidity test.
If the test is positive, hemoglobin electrophoresis is required to distinguish between children who
have the genetic trait and children who have the disease.
354. A nurse is assessing a school-age child who has meningitis. Which of the following findings
is the priority for the nurse to report to the provider?
Answer: Petechiae on the lower extremities

The presence of a petechial or purpuric rash on a child who is ill can indicate the presence of
meningococcemia. This type of rash indicates the greatest risk of serious rapid complications
from sepsis and should be reported immediately to the provider.
355. A nurse is assessing an infant who has a ventricular septal defect.
Which of the following findings should the nurse expect?
Answer: Loud, harsh murmur
The nurse should expect to hear a loud, harsh murmur with a ventricular septal defect due to the
left-to-right shunting of blood, which contributes to hypertrophy of the infant's heart muscle.
356. A nurse is creating a plan of care for an infant who has an epidural hematoma from a head
injury. Which of the following interventions should the nurse include in the plan?
Answer: Implement seizure precautions for the infant.
An infant who has an epidural hematoma is at great risk for seizure activity. Therefore, the nurse
should implement seizure precautions for the child.
357. A nurse is caring for an adolescent who received a kidney transplant. Which of the
following findings should the nurse identify as an indication the adolescent is rejecting the
kidney?
Answer: Serum creatinine 3.0 mg/dL
Creatinine is a byproduct of protein metabolism and is excreted from the body through the
kidneys. An elevated serum creatinine level, therefore, can be an indication that the kidneys are
not functioning. The nurse should identify that the adolescent's serum creatinine level is higher
than the expected reference range of 0.4 to 1.0 mg/dL for an adolescent and can indicate
rejection of the kidney.
358. A nurse in an emergency department is performing an admission assessment on a 2 weekold male newborn. Which of the following findings is the priority for the nurse to report to the
provider?
Answer: Substernal retractions

When using the airway, breathing, and circulation approach to client care, the nurse should
determine that the priority finding to report to the provider is substernal retractions. This finding
indicates the newborn is experiencing increased respiratory effort, which could quickly progress
to respiratory failure.
359. A hospice nurse is caring for a preschooler who has a terminal illness. The father tells the
nurse that he cannot cope anymore and has decided to move out of the house. Which of the
following statements should the nurse make?
Answer: "Let's talk about some of the ways you have handled previous stressors in your life."
This statement offers a general lead to allow the parent to express their feelings and previous
actions when faced with stressful situations. It also helps the parent to focus on ways that they
can cope with the current situation.
360. A nurse in an emergency department is caring for an adolescent who has severe abdominal
pain due to appendicitis. Which of the following locations should the nurse identify as
McBurney's point?
Answer: The nurse should identify this area of the client's abdomen as McBurney's point. This
area of the right lower quadrant located about two-thirds of the way between the umbilicus and
the client's anterosuperior iliac spine is the area where a client who has appendicitis is most
likely to report pain and tenderness.
361. A nurse is reviewing the laboratory report of a 7 year-old child who is receiving
chemotherapy. Which of the following lab values should the nurse report to the provider?
Answer: Hgb 8.5 g/dL
A child receiving chemotherapy is at risk for anemia due to the chemotherapy effects on the
blood-forming cells of the bone marrow. The development of anemia is diagnosed through
laboratory testing of hemoglobin and hematocrit levels. The nurse should recognize that a
hemoglobin level of 8.5 g/dL is below the expected reference range of 10 to 15.5 g/dL for a 7year-old child and should be reported to the provider.

362. A nurse is caring for a 15 year-old client who is married and is scheduled for a surgical
procedure. The client asks, "who should sign my surgical consent?" Which of the following
responses should the nurse make?
Answer: "You can sign the consent form because you are married."
The nurse should inform the adolescent that marriage gives adolescents the legal right to consent
to surgical procedures and sign other legal documents that they would not otherwise be able to
sign due to their age.
363. A nurse is assessing a 4-year-old child at a well-child visit. Which of the following
developmental milestones should the nurse expect to observe?
Answer: Cuts an outlined shape using scissors.
The nurse should recognize that an expected developmental milestone of a 4-year-old child is
using scissors to cut out a shape.
364. A nurse is caring for an infant who has respiratory syncytial virus (RSV). Which of the
following actions should the nurse implement for infection control?
Answer: Have a designated stethoscope in the infant's room.
The nurse should initiate droplet precautions for an infant who has RSV because the virus is
spread by direct contact with respiratory secretions. Therefore, designated equipment, such as a
blood pressure cuff and a stethoscope, should be placed in the infant's room.
365. A nurse in an emergency department is caring for a school-age child who has appendicitis
and rates their abdominal pain as 7 on a scale of 0 to 10. Which of the following actions should
the nurse take?
Answer: Give morphine 0.05mg/kg IV
A pain level of 7 on a scale of 0 to 10 is considered severe. The nurse should administer an
analgesic medication for pain relief.
366. A nurse is assessing the vital signs of a 10-year-old child following a burn injury. The nurse
should identify that which of the following findings in an indication of early septic shock?
Answer: Temperature 39.1°C (102.4°F)

The nurse should identify that a temperature of 39.1°C (102.4°F) is above the expected reference
range of 37° to 37.5°C (98.6° to 99.5°F) for a 10-year-old child. The nurse should expect a child
who has early septic shock to have a fever and chills.
367. A school nurse is assessing an adolescent who has multiple burns in various stages of
healing. Which of the following behaviors should the nurse identify as a possible indication of
physical abuse?
Answer: Denies discomfort during assessment of injuries.
The nurse should suspect child maltreatment in the form of physical abuse if the adolescent has a
blunted response to painful stimuli or injury.
368. A nurse is caring for a 15 year-old client following a head injury. Which of the following
findings should the nurse identify as an indication that the child is developing syndrome of
inappropriate antidiuretic hormone secretion (SIADH)?
Answer: Mental confusion
A child who has a head injury can develop SIADH as a result of altered pituitary function,
leading to an oversecretion of antidiuretic hormone. Oversecretion of antidiuretic hormone leads
to a decrease in urine output, hyponatremia, and hypoosmolality due to overhydration. As the
hyponatremia becomes more severe, mental confusion and other neurologic manifestations such
as seizures can occur.
369. A nurse is caring for a toddler who has spastic (pyramidal) cerebral palsy. Which of the
following findings should the nurse expect? (Select all that apply.)
Answer: • Ankle clonus
• Exaggerated stretch reflexes
• Contractures
370. A nurse in a provider's office if preparing to administer immunizations to a toddler during a
well-child visit. Which of the following actions should the nurse plan to take?
Answer: Withhold the measles, mumps, and rubella (MMR) vaccine.

The nurse should recognize that an allergy to neomycin with an anaphylactic reaction is a
contraindication for receiving the MMR vaccine. Clients who have a severe allergy to eggs or
gelatin should not receive this vaccine.
371. A school nurse is assessing an adolescent who has scoliosis. Which of the following
findings should the nurse expect?
Answer: A unilateral rib hump
When assessing an adolescent for scoliosis, the school nurse should expect to see a unilateral rib
hump with hip flexion. This results from a lateral S- or C-shaped curvature to the thoracic spine
resulting in asymmetry of the ribs, shoulders, hips, or pelvis. Scoliosis can be the result of a
neuromuscular or connective tissue disorder, or it can be congenital in nature.
372. A nurse is caring for a preschooler whose father is going home for a few hours while
another relative stays with the child. Which of the following statements should the nurse make to
explain to the child when their father will return?
Answer: "Your daddy will be back after you eat."
Preschoolers make sense of time best when they can associate it with an expected daily routine,
such as meals and bedtime. Therefore, the child comprehends time best when it is explained to
them in relation to an event they are familiar with, such as eating.
373. The nurse is caring for a preschooler who has been receiving IV fluids via a peripheral IV
catheter. When preparing to discontinue the IV fluids and catheter, which of the following
actions should the nurse plan to take?
Answer: First, the nurse should turn off the IV pump. Next, the nurse should occlude the IV
tubing, and then remove the tape securing the catheter. Last, the nurse should apply pressure over
the catheter insertion site.
374. A nurse is caring for a school-age child who has experienced a tonic-clonic seizure. Which
of the following actions should the nurse take during the immediate postictal period?
Answer: Place the child in a side-lying position.
The nurse should place the child in a side-lying position to prevent aspiration.

375. A nurse is teaching the guardian of a 6-month-old infant about car seat use. Which of the
following statements by the guardian indicates an understanding of the teaching?
Answer: "I should secure the car seat using lower anchors and tethers instead of the seat belt."
Lower anchors and tethers, or the LATCH child safety seat system, should be used to secure an
infant's car seat in the vehicle. This system provides anchors between the front cushion and the
back rest for the car seat. Therefore, if this system is available, the seat belt does not have to be
used.
376. A nurse in an urgent care clinic is assessing an adolescent who has an upper respiratory tract
infection. Which of the following findings should the nurse identify as a manifestation of
pertussis?
Answer: Dry, hacking cough
The nurse should identify that a dry, hacking cough is a manifestation of pertussis. This disease
usually begins with indications of an upper respiratory tract infection, which includes a dry,
hacking cough that is sometimes more severe at night.
377. A nurse is preparing to administer an immunization to a 4-year-old child. Which of the
following actions should the nurse plan to take?
Answer: Administer the immunization using a 24-gauge needle.
The nurse should administer an immunization for a 4-year-old child using a 22- to 25- gauge
needle to minimize the amount of pain the child experiences.
378. The nurse is providing discharge teaching to the parent of a child who is 1 week
postoperative following a cleft palate repair. For which of the following members of the inter
professional team should the nurse initiate a referral?
Answer: Speech therapist
The nurse should initiate a referral for a speech therapist for a child who is postoperative
following a cleft palate repair. A child who has a cleft palate will require speech therapy
immediately following the repair to support speech development and future articulation.

379. A school nurse is preparing to administer atomoxetine 1.2 mg/kg/day PO to a school aged
child who weights 75 lbs. Available is atomoxetine 40 mg/capsule. How many capsules should
the nurse administer per day?
Answer: 1 capsule
380. A nurse is caring for a school-age child who is receiving a blood transfusion. Which of the
following manifestations should alert the nurse to a possible hemolytic transfusion reaction?
Answer: Flank pain
The nurse should recognize that flank pain is caused by the breakdown of RBCs and is an
indication of a hemolytic reaction to the blood transfusion.
381. A nurse in the emergency department is caring for a toddler who has a partial thickness
burns on their right arm. Which of the following actions should the nurse take?
Answer: Cleanse the affected area with mild soap and water.
The nurse should wash the affected area with mild soap and water to remove any loose tissue that
could cause infection.
382. A nurse is auscultating the lungs of an adolescent who has asthma. The nurse should
identify the sound as which of the following?
Answer: Tachypnea
The nurse should identify the sound heard during auscultation as tachypnea, which is a rapid,
regular breathing pattern. This breathing pattern often occurs with anxiety, fever, metabolic
acidosis, or severe anemia.
383. A nurse is reviewing the lumbar puncture results of a school-age child suspected of having
bacterial meningitis. Which of the following results should the nurse identify as a finding
associated with bacterial meningitis?
Answer: Increased protein concentration
The nurse should identify that an increased protein concentration in the spinal fluid is a finding
that can indicate bacterial meningitis.

384. A nurse is assessing a 3-year-old toddler at a well-child visit. Which of the following
manifestations should the nurse report to the provider?
Answer: Respiratory rate 45/min
The nurse should identify that a respiratory rate of 45/min is above the expected reference range
of 20 to 25/min for a 3-yearold toddler and can indicate respiratory dysfunction and acute
respiratory distress. Therefore, the nurse should report this finding to the provider.
385. A nurse is teaching the parents of a school-age child who has a new diagnosis of
osteomyelitis of the tibia. The nurse should identify that which of the following statements by
the parents indicates an understanding of the teaching?
Answer: "My child will receive antibiotics for several weeks."
The nurse should instruct the parent that the child will receive antibiotic therapy for at least 4
weeks. Surgery might be indicated if the antibiotics are not successful.
386. A nurse is providing teaching about social development to the parents of a preschooler.
Which of the following play activities should the nurse recommend for the child?
Answer: Playing dress-up
The nurse should instruct the parents that at the preschool age, play should focus on social,
mental, and physical development. Therefore, playing dress-up is a recommended play activity
for this child.
387. A nurse is reviewing the laboratory report of an infant who is receiving treatment for severe
dehydration. The nurse should identify that which of the following laboratory values indicates
effectiveness of the current treatment?
Answer: Sodium 140 mEq/L
The nurse should identify that a sodium level of 140 mEq/L is within the expected reference
range of 134 to 150 mEq/L and indicates the current treatment regimen the infant is receiving for
dehydration is effective.

388. A nurse is caring for a preschooler who is scheduled for hydrotherapy treatment for wound
debridement following a burn injury. Which of the following actions should the nurse take prior
to the procedure?
Answer: Administer an analgesic to the child.
Hydrotherapy for debridement of a wound is an extremely painful procedure which requires
analgesia and/or sedation. When pain is controlled, it leads to reduced physiological demands on
the body caused by stress and decreases the likelihood of children developing depression and
post-traumatic stress disorder.
389. A charge nurse in an emergency department is preparing an inservice for a group of newly
licensed nurses on the clinical manifestations of child maltreatment. Which of the following
manifestations should the charge nurse include as suggestive of potential physical abuse?
Answer: Symmetric burns of the lower extremities
The nurse should include that symmetric burns to the lower extremities can indicate physical
abuse. The patterns are usually characteristic of the method or object used, such as cigar or
cigarette burns, or burns in the shape of an iron.
390. A nurse is caring for a school-age child who in in Buck's traction following a leg fracture 24
hr ago. Which of the following actions should the nurse take?
Answer: Assess peripheral pulses once every 4 hr.
Buck's traction is a type of skin traction that can be used to immobilize extremities prior to
surgery. The nurse should provide frequent neurovascular checks at least every 4 hr after the first
24 hr of placement in Buck's traction. The nurse should monitor and report signs of
neurovascular impairment in the extremities such as cyanosis, edema, pain, absent pulses, and
tingling.
391. A nurse is planning care for a toddler who has a serum lead level of 4 mcg/dL. Which of the
following actions should the nurse plan to take?
Answer: Schedule the toddler for a yearly rescreening.
The nurse should schedule the toddler for a lead level rescreening in 1 year and educate the
family on ways to prevent exposure.

392. A nurse is receiving change-of-shift report on four children. Which of the following children
should the nurse see first?
Answer: A school-age child who has sickle cell anemia and reports decreased vision in the left
eye.
When using the urgent vs. nonurgent approach to client care, the nurse should determine the
priority finding is a report of decreased vision in the left eye. This finding indicates that the child
is experiencing a vaso-occlusive crisis and should be reported to the provider immediately.
Therefore, the nurse should see this child first.
393. A nurse is providing teaching to the parent of an infant who has diaper dermatitis. The nurse
should instruct the parent to apply which of the following to the affected area?
Answer: Zinc oxide
Diaper dermatitis is a common inflammatory skin disorder caused by contact with an irritant
such as urine, feces, soap, or friction, and takes the form of scaling, blisters, or papules with
erythema. Providing a protective barrier, such as zinc oxide, against the irritants allows the skin
to heal.
394. A nurse is caring for a school-age child who is receiving chemotherapy and is severely
immunocompromised. Which of the following actions should the nurse take?
Answer: Screen the child's visitors for indications of infection.
A child who is severely immunocompromised is unable to adequately respond to infectious
organisms, resulting in the potential for overwhelming infection. Therefore, the nurse should
screen the child's visitors for indications of infection.
395. A nurse is assessing a toddler who has gastroenteritis and is exhibiting manifestations of
dehydration. Which of the following findings is the nurses priority?
Answer: Tachypnea
When using the airway, breathing, and circulation approach to client care, the nurse's priority
finding is the toddler's tachypnea. Tachypnea is a result of the kidneys being unable to excrete
hydrogen ions and produce bicarbonate, which leads to metabolic acidosis.

396. A nurse is teaching a school-age child who has a new diagnosis of type 1 diabetes mellitus.
Which of the following statements by the child indicates an understanding of the teaching?
Answer: "I will give myself a shot of regular insulin 30 minutes before I eat breakfast."
The child should administer regular insulin 30 min before meals so that the onset coincides with
food intake.
397. A nurse is teaching the parent of an infant about ways to prevent sudden infant death
syndrome (SIDS). Which of the following instructions should the nurse include?
Answer: "Give the infant a pacifier at bedtime."
The nurse should inform the parent that protective factors against SIDS include breastfeeding
and the use of a pacifier when the infant is sleeping.
398. The nurse is assessing a school-age child who has peritonitis. Which of the following
findings should the nurse expect?
Answer: Abdominal distension
The nurse should identify that abdominal distention is an expected finding of peritonitis.
Peritonitis is an inflammation of the lining of the abdominal wall. This inflammation in the
abdomen, along with the ileus that develops, causes abdominal distention. Other manifestations
include chills, irritability, and restlessness.
399. The nurse is interviewing the parent of an 18-month-old toddler during a well-child visit.
The nurse should identify that which of the following findings indicates a need to assess the
toddler for hearing loss?
Answer: The toddler received tobramycin during a hospitalization 2 weeks ago.
The nurse should identify tobramycin as an aminoglycoside, which is an ototoxic medication that
can cause mild to moderate hearing loss, and should assess the toddler for a hearing impairment.
400. A nurse is providing teaching to the parent of a school-age child who has a new prescription
for oral nystatin for the treatment of oral candidiasis. Which of the following instructions should
the nurse include?

Answer: "Shake the medication prior to administration."
The nurse should instruct the parent to shake the medication prior to administration to disperse
the medication evenly within the suspension.
401. A nurse is admitting a school-age child who has Pertussis. Which of the following actions
should the nurse take?
Answer: Initiate droplet precautions for the child.
The nurse should initiate droplet precautions for a child who has pertussis, also known as
whooping cough. Pertussis is transmitted through contact with infected large-droplet nuclei that
are suspended in the air when the child coughs, sneezes, or talks.
402. A nurse is caring for a toddler who is experiencing acute diarrhea and has moderate
dehydration. Which of the following nutritional items should the nurse offer to the toddler?
Answer: Oral rehydration solution
A toddler who has acute diarrhea should consume an oral rehydration solution to replace
electrolytes and water by promoting the reabsorption of water and sodium. This promotes
recovery from dehydration.
403. A nurse in an emergency department is caring for a school-age child who is experiencing an
anaphylactic reaction. Which of the following is the priority action by the nurse?
Answer: Administer epinephrine IM to the child.
When using the urgent vs. nonurgent approach to client care, the nurse should determine that the
priority action is administering epinephrine IM to the child. During an anaphylactic reaction,
histamine release causes bronchoconstriction and vasodilation. This is an emergency because
ultimately this causes decreased blood return to the heart.
404. A nurse is planning care for a newly admitted schole-age child who has generalized seizure
disorder. Which of the following interventions should the nurse plan to include?
Answer: Ensure the oxygen source is functioning in the childs room: The nurse should recognize
that maintaining the child's airway is important during a seizure. The nurse should ensure that the

oxygen source is functioning because the child might require supplemental oxygen following a
seizure.
405. A nurse is providing dietary teaching to the guardian of a schoolage child who has cystic
fibrosis. Which of the following statements should the nurse make?
Answer: "You should offer your child high-protein meals and snacks throughout the day." The
nurse should instruct the guardian to provide a diet that is well-balanced and high in protein and
calories. Children who have cystic fibrosis require a higher percentage of the recommended
dietary allowances of all nutrients to meet their energy requirements. Children who have good
nutritional intake have improved lung function and decreased risk of infection.
406. A nurse is providing discharge teaching to the parents of a 6month-old infant who is
postoperative following hypospadias repair with a stent placement. Which of the following
instructions should the nurse include in the teaching?
Answer: "Allow the stent to drain into your infants diaper." The nurse should instruct the parents
to ensure that the stent drains directly into the infant's diaper to prevent kinking or twisting that
can interfere with urine flow.
407. A nurse is caring for a school-age child who has primary nephrotic syndrome and is taking
prednisone. Following 1 week of treatment, which of the following manifestations indicates to
the nurse that the medication is effective?
Answer: Decreased edema: A child who has nephrotic syndrome can experience edema due to
the increased glomerular permeability, which increases protein loss. Prednisone decreases
glomerular permeability, which causes fluid to shift from the extracellular spaces, resulting in
decreased edema.
408. A nurse is receiving change-of-shift report for four children. Which of the following
children should the nurse assess first?
Answer: A toddler who has a concussion and an episode of forceful vomiting. When using the
urgent vs. nonurgent approach to client care, the nurse should assess this child first. An episode

of forceful vomiting is an indication of increased intracranial pressure in a toddler who has a
concussion.
409. A nurse is providing discharge teaching to the guardians of a toddler who had lower leg cast
applied 24 hr ago. The nurse should instruct the guardians to report which of the following
finding to the provider?
Answer: Restricted ability to move the toes: The nurse should inform the guardians that a
restricted ability of the toddler to move their toes is an indication of neurovascular compromise
and requires immediate notification of the provider. Permanent muscle and tissue damage can
occur in just a few hours.
410. A nurse in an emergency department is auscultating the lungs of an adolescent who is
experiencing dyspnea. The nurse should identify the sound as which of the following?
Answer: Wheezes: The nurse should identify the sound during auscultation as wheezes, which
are high-pitched, musical or whistling-like sounds heard primarily on expiration as air passes
through and vibrates narrowed airways.
411. A nurse is caring for a preschooler who has congestive heart failure. The nurse observes
wide QRS complexes and peaked T waves on the cardiac monitor. Which of the following
prescriptions should the nurse clarify with the provider?
Answer: Potassium Chloride: The nurse should identify that a child who has congestive heart
failure can develop electrolyte imbalances, such as hyperkalemia or hypokalemia. The nurse
should identify that the child is exhibiting manifestations of hyperkalemia and contact the
provider about the administration of potassium chloride, which can increase the severity of
hyperkalemia.
412. A nurse is planning an educational program for school-age children and their parents about
bicycle safety. Which of the following information should the nurse plan to include?
Answer: The child should be able to stand on the balls of their feet when sitting on the bike: To
decrease the risk for injury, parents should ensure that the bike is the correct size for the child.
When seated on the bike, the child should be able to stand with the ball of each foot touching the

ground and should be able to stand with each foot flat on the ground when straddling the bike's
center bar.
413. A nurse is monitoring the oxygen saturation level of an infant using pulse oximetry. The
nurse should secure the sensor to which of the following areas on the infant?
Answer: Great Toe. The nurse should secure the sensor to the great toe of the infant and then
place a snug-fitting sock on the foot to hold the sensor in place. The nurse should also check the
skin under the sensor site frequently for temperature, color, and the presence of a pulse.
414. A nurse is an emergency department is caring for a school-age child who has epiglottitis.
Which of the following actions should the nurse take?
Answer: Monitor the childs oxygen saturation: The nurse should monitor the child's oxygen
saturation level because the child is experiencing acute respiratory distress and it is necessary to
determine if the child is responding to treatment.
415. A nurse in an emergency department is caring for a school-age child who has sustained a
minor superficial burn from fireworks on their forearm. Which of the following actions should
the nurse take?
Answer: Apply an antimicrobial ointment to the affected area: The nurse should apply an
antimicrobial ointment to the burned area to prevent infection.
416. A nurse in a providers office is caring for a school-age child who has varicella. The parents
asks the nurse when their child will no longer be contagious. Which of the following responses
should the nurse make?
Answer: "When your childs lesions are crusted, usually 6 days after they appear.": The nurse
should inform the parent that the child is contagious 1 day prior to lesion eruption and until the
vesicles have crusted over, which usually takes about 6 days.
417. A nurse is providing discharge teaching to the parent of a school age child who has
moderate persistant asthma. Which of the following instructions should the nurse include?

Answer: "Pulmonary function tests will be performed every 12 to 24 months to evaluate how
your child is responding to therapy.": The nurse should inform the parent that their child will
need pulmonary function tests every 12 to 24 months to evaluate the presence of lung disease
and how the child is responding to the current treatment regimen. As children grow, sometimes
their manifestations can improve or decline, and treatment needs to change accordingly.
418. A nurse is admitting an infant who has intussusception. Which of the following findings
should the nurse expect? (Select all that apply.)
Answer: Vomiting
Lethargy Vomiting is correct. The nurse should expect an infant who has intussusception to
exhibit vomiting due to the obstruction that occurs when a segment of the bowel telescopes
within another segment of the bowel. Lethargy is correct. The nurse should expect an infant who
has intussusception to exhibit lethargy due to episodes of severe pain during which the infant
cries inconsolably, leading to exhaustion and decreased nutritional intake.
419. A nurse is reviewing the laboratory results of a school-age child who is 1 week
postoperative following an open fracture repair. Which of the following findings should the
nurse identify as an indication of a potential complication?
Answer: Erythrocyte sedimentation rate 18 mm/hr: The nurse should identify that an erythrocyte
sedimentation rate of 18 mm/hr is above the expected reference range of up to 10 mm/hr and is
an indication of osteomyelitis.
420. A nurse is providing discharge teaching to the parents of a 3month old infant following a
cheiloplasty. Which of the following instructions should the nurse include?
Answer: "Apply a thin layer of antibiotic ointment on the your baby’s suture line daily for the
next 3 days.": The nurse should instruct the parents to apply a thin layer of antibiotic ointment on
the infant's suture line daily for 3 days and then continue to apply petroleum jelly to the area for
several weeks to promote healing.

421. A nurse is discussion organ donation with the parents of a school- age child who has
sustained brain death due to a bicycle crash. Which of the following actions should the nurse
take first?
Answer: Explore the parents feelings and wishes regarding organ donation: The first action the
nurse should take when using the nursing process is assessment. The nurse should first explore
the parents' feelings and wishes regarding organ donation to assist in determining if organ
donation is the right choice for the family.
422. A nurse is caring for a 1-month-old infant who is breastfeeding and requires a heel stick.
Which of the following actions should the nurse take to minimize the infants pain?
Answer: Allow the mother to breastfeed while the sample is being obtained: The nurse should
allow the mother to breastfeed the infant prior to or during the procedure. Evidence-based
practice indicates breastfeeding or non-nutritive sucking with a pacifier can provide
nonpharmacological pain management in infants.
423. A nurse is assessing an adolescent who received a sodium polystyrene sulfonate enema.
Which of the following findings indicates effectiveness of the medication?
Answer: Serum potassium level 4.1 mEq/L, The nurse should monitor the adolescent's serum
potassium level following the administration of sodium polystyrene sulfonate. This medication is
used to treat hyperkalemia by exchanging sodium ions for potassium ions in the intestine.
Therefore, a potassium level within the expected reference range of 3.4 to 4.7 mEq/L indicates
the effectiveness of the medication.
424. A charge nurse is preparing to make a room assignment for a newly admitted school-age
child. Which of the following considerations is the nurses priority?
Answer: Disease process: The transmission of infectious diseases is the greatest risk to this child
and other children on the unit. Therefore, the child's disease process is the nurse's priority
consideration
425. A nurse is assessing the pain level of a 3-year-old toddler. Which of the following pain
assessment scales should the nurse use?

Answer: FACES: The nurse should use the FACES pain rating scale for pediatric clients who are
3 years old and older. This scale allows the toddler to point to the face that depicts their current
level of pain. The nurse can then determine the need for pain management.
426. A nurse is preparing to administer ibuprofen 5 mg/kg every 6 hr PRN for a temperatures
above 38.0C (100.5F) to an infant who weighs 17.6 lb. Available is ibuprofen oral suspension
100mg/5mL. How many mL should the nurse administer to the infant per dose?
Answer: 2 mL
427. A nurse is assessing a 6-month-old infant during a well-child visit. Which of the following
findings should the nurse report to the provider?
Answer: Presence of strabismus: Strabismus, or crossing of the eyes, typically disappears at 3 to
4 months of age. If not corrected early, this can lead to blindness. Therefore, the nurse should
report this finding to the provider
428. A school nurse is caring for a child following tonic-clonic seizure. Which of the following
actions should the nurse take first?
Answer: Check the childs respiratory rate.: When using the airway, breathing, and circulation
approach to client care, the nurse should determine the priority action is to assess the child's
respiratory rate. If the child is not breathing, the nurse should administer rescue breaths.
429. A nurse is planning developmental activities for a newly admitted 10-year-old child who has
neutropenia. Which of the following actions should the nurse plan to take?
Answer: Provide the child with a book about adventure : The nurse should provide a school-age
child with a book about adventure as a developmental activity because children are expanding
their knowledge and imagination during this age. Through reading, school-age children can feel
powerful and skillful as they imagine themselves in the stories they read.
430. A nurse in a health department is caring for an emancipated adolescent who has an STI and
is unaccompanied by a guardian. Which of the following actions should the nurse take?

Answer: Have the adolescent sign a consent form for treatment: The nurse should identify that
an emancipated minor can sign the consent form for treatment of an STI or any other form of
medical treatment requiring consent.
431. A nurse is assessing an 8-year-old child who has early indications of shock. After
establishing an airway and stabilizing the childs respirations, which of the following actions
should the nurse take next?
Answer: Initiate IV access: After establishing an airway and stabilizing the child's respirations,
the next action the nurse should take when using the airway, breathing, and circulation approach
to client care is to establish IV access to maintain the child's circulatory volume.
432. A nurse is performing hearing screenings for children at a community health fair. Which of
the following children should the nurse refer to a provider for a more extensive hearing
evaluation?
Answer: An 8-month-old who is not yet making babbling sounds: The nurse should refer an
infant who is not making babbling sounds by the age of 7 months to a provider for a more
extensive evaluation of hearing
433. A nurse is providing discharge teaching to the guardian of a school-age child who has
undergone a tonsillectomy. Which of the following statements by the guardian indicates an
understanding the teaching?
Answer: "I will notify the doctor if I notice that my child is swallowing frequently."
434. A community health nurse is assessing an 18-month-old toddler in a community day care.
Which of the following findings should the nurse identify as a potential indication of physical
neglect?
Answer: Poor personal hygiene: A toddler who has poor personal hygiene can be a potential
indication of physical neglect. Because toddlers are still dependent on their parents or guardians
for help with hygiene needs, poor personal hygiene can indicate a lack of supervision.

435. A nurse assessing a school-age child who has an infratentorial brain tumor. Which of the
following findings should the nurse identify as a manifestation of increased intracranial
pressure?
Answer: Difficulty concentrating: The nurse should identify that irritability, inability to follow
commands, and difficulty concentrating are manifestations of increased intracranial pressure due
to decreased blood flow within the brain and pressure on the brainstem.
436. A nurse is providing teaching to an adolescent about how to manage tinea pedis. Which of
the following statements by the adolescent indicates an understanding of the teaching?
Answer: "I should wear sandals as much as possible.": Sandals allow air to circulate around the
feet, decreasing perspiration and eliminating the medium for bacteria and fungus to grow. The
nurse should inform the adolescent that wearing sandals, open-toed, or well-ventilated shoes will
promote healing of the fungal infection.
437. A nurse is caring for a school-age child who has diabetes mellitus and was admitted with a
diagnosis of diabetic ketoacidosis. When performing the respiratory assessment, which of the
following findings should the nurse expect?
Answer: Deep respirations of 32/min: The nurse should expect Kussmaul respirations in a child
who has diabetic ketoacidosis. These deep and rapid respirations are the body's attempt to
eliminate excess carbon dioxide and achieve a state of homeostasis.
438. A nurse is planning an educational program to teach parents about protecting their children
from sunburns. Which of the following instructions should the nurse plan to include?
Answer: "Choose a waterproof sunscreen with a minimum SPF of 15.": The nurse should
instruct parents to apply a waterproof sunscreen with a minimum SPF of 15 for children. The
parents should apply the sunscreen prior to sun exposure to reduce the risk of sunburn.
439. A nurse is providing teaching to the parents of a preschooler who has heart failure and a
new prescription for digoxin twice daily. Which of the following instructions should the nurse
include in the teaching?

Answer: "Brush the child’s teeth after giving the medication.": The nurse should teach the parent
to closely monitor the child's number of wet diapers. Monitoring the number of wet diapers per
day is an effective way for the parent to monitor adequate output and hydration status.
440. A nurse is providing teaching to the family of a school-age child who has juvenile idiopathic
arthritis. Which if the following instructions should the nurse include in the teaching?
Answer: "Encourage the child to perform independent self-care.": The nurse should teach the
family the importance of encouraging the child to perform independent selfcare. This will
minimize the child's pain while maximizing mobility. Encouraging and praising the child's
efforts for independence will also increase their self-esteem.
441. A nurse is creating a plan of care for a child who has varicella. Which of the following
interventions should the nurse include?
Answer: Initiate airborne precautions for the child: The nurse should initiate airborne
precautions for a child who has varicella because it is spread through droplets in the air. The
incubation period for varicella is 2 to 3 weeks, and the child is contagious even before lesions
appear.
442. A nurse is assessing a school-age child who has an acute spinal cord injury following a
sports injury 1 week ago. Identify the area the nurse should tap to elicit the bicep reflect.
Answer: To elicit the bicep reflex in a school-age child, the nurse should tap the biceps tendon,
located in the antecubital fossa (the crease of the elbow). This reflex is tested by placing a finger
over the biceps tendon and tapping it with a reflex hammer.
443. A school nurse is providing an in-service for faculty about improving education for students
who have ADHD. Which of the following statements by a faculty member indicates an
understanding of the teaching?
Answer: "I will teach challenging academic subjects to students who have ADHD in the
morning." Faculty should plan to teach challenging academic subjects in the morning when
students who have ADHD are most able to focus and their medication is most likely to be
effective

444. A nurse is caring for a school-age child who has peripheral edema. The nurse should
identify that which of the following assessments should be performed to confirm peripheral
edema?
Answer: Palpate the dorsum of the child’s feet: The nurse should palpate the dorsum of the feet
by pressing the fingertip against a bony prominence for 5 seconds to assess for peripheral edema.
445. A nurse is caring for an infant who is receiving IV fluids for the treatment of Tetralogy of
Fallot and begins to have hyperkeratotic spell. Which of the following actions should the nurse
take?
Answer: Place the infant in a knee-chest position: The nurse should place the infant in a kneechest position during a hypercyanotic spell to decrease the return of desaturated venous blood
from the legs and to direct more blood into the pulmonary artery by increasing systemic vascular
resistance.
446. A nurse is reviewing the dietary choices of an adolescent who has iron deficiency anaemia.
The nurse should identify that which of the following menu items has the highest amount of
nonheme iron?
Answer: ½ cup raisins: The nurse should encourage the adolescent to eat raisins because they
contain the highest amount of nonheme iron.
447. A nurse in an emergency department is assessing a 3-month-old infant who has rotavirus
and is experiencing acute vomiting and diarrhoea. Which of the following manifestations should
the nurse identify as an indication that the infant has moderate to severe dehydration?
Answer: Sunken anterior fontanel: The nurse should recognize that a sunken anterior fontanel is
an indication of moderate to severe dehydration due to the acute loss of fluid.
448. A nurse is planning care for a school-age child who has tunnelled central venous access
device. Which of the following interventions should the nurse include in the plan?
Answer: Use a semipermeable transparent dressing to cover the site: The nurse should cover the
site with a semipermeable transparent dressing to reduce the risk of infection.

449. A nurse is teaching a group of parents about infectious mononucleosis. Which of the
following statements by parent indicates an understanding the teaching?
Answer: "Mononucleosis is caused by an infection with the Epstein-Barr virus.": The nurse
should identify that mononucleosis is a mildly contagious illness that occurs sporadically or in
groups, and is primarily caused by the Epstein-Barr virus.
450. A nurse is caring for a newly admitted school-age child who has hypopituitarism. Which of
the following medications should the nurse expect the provider to prescribe?
Answer: Recombinant growth hormone: Recombinant growth hormone injections are used to
treat hypopituitarism, which inhibits cell growth and results in growth failure. The nurse should
expect the provider to prescribe this treatment.
451. A nurse is creating a plan of care for a preschooler who has Wilms' tumour and is scheduled
for surgery. Which of the following interventions should the nurse include?
Answer: Avoid palpating the abdomen when bathing the child before surgery: The nurse should
avoid palpating the abdomen when bathing the child before surgery because movement of the
tumour can cause cancer cells to disseminate to other sites, adjacent and distant to the tumour
site.
452. A nurse is providing discharge teaching to the parent of an 18month-old toddler who has
dehydration due to acute diarrhoea. Which of the following statements by the parent indicates an
understanding of the teaching?
Answer: "I will monitor my child’s number of wet diapers.": The nurse should teach the parent
to closely monitor the child's number of wet diapers. Monitoring the number of wet diapers per
day is an effective way for the parent to monitor adequate output and hydration status.
453. A nurse is teaching the guardian of a 6-month-old infant about teething. Which of the
following statements should the nurse make?
Answer: "Your baby might pull at their ears when they are teething."

454. A nurse is creating a plan of care for a newly admitted adolescent who has bacterial
meningitis. How long should the nurse plan to maintain the adolescent in droplet precautions?
Answer: For 24 hr following initiation of antimicrobial therapy: The nurse should plan to
maintain the adolescent on droplet precautions for at least 24 hr following initiation of
antimicrobial therapy. This practice will ensure that the adolescent is no longer contagious,
which protects family members and the personnel caring for the client. Prophylactic antibiotics
might be prescribed to individuals who were in close contact with the adolescent.
455. A nurse is providing anticipatory guidance to the parent of a toddler. Which of the following
expected behavior characteristics of toddlers should the nurse include?
Answer: Expressed likes and dislikes: The nurse should include that expressing likes and
dislikes is an expected behavior of toddlers. This is the time in life when a toddler is developing
autonomy and self-concept. They will try to assert themselves and frequently refuse to comply.
The parent should allow the child to have some control, but also set limits for them so they learn
from their behavior and learn to control their actions.
456. A nurse is admitting a 4-month-old infant who has heart failure. Which of the following
findings is the nurses priority?
Answer: Episodes of vomiting: When using the urgent vs. nonurgent approach to client care, the
nurse should determine that the priority finding is three episodes of vomiting. This can indicate
digoxin toxicity, which requires immediate intervention. Therefore, this is the nurse's priority
finding.
457. A nurse in an emergency department is assessing a toddler who has Kawasaki disease.
Which of the following findings should the nurse expect? (Select all that apply.)
Answer: Increased temperature
Xerophthalmia
Cervical lymphadenopathy Increased temperature is correct. Kawasaki disease is an acute illness
associated with a fever that is unresponsive to antipyretics or antibiotics. Gingival hyperplasia is
incorrect. Children who have Kawasaki disease develop a strawberry tongue, cracked lips, and
edema of the oral mucosa and pharynx. A child who is receiving phenytoin therapy can develop

gingival hyperplasia. Xerophthalmia is correct. Ophthalmic manifestations of Kawasaki disease
include reddening of the conjunctiva and dryness of the eyes, or xerophthalmia. Bradycardia is
incorrect. Kawasaki disease is an infection that affects the vascular system, including the heart.
The nurse should expect the child to be tachycardic with a gallop rhythm. Long-term effects of
Kawasaki disease include the development of coronary artery aneurysms or myocardial
infarction. Cervical lymphadenopathy is correct. A child who has Kawasaki disease can develop
enlarged cervical nodes on one side of the neck that are nontender and greater than 1.5 cm in
size.
458. A nurse is caring for a 10-year-old child following a head injury. Which of the following
findings should the nurse identify as an indication that the child is developing diabetes insipidus?
Answer: Sodium 155 mEq/L: A child who has a head injury can develop diabetes insipidus as a
result of pituitary hypofunction leading to a deficiency of antidiuretic hormone. Underexcretion
of antidiuretic hormone leads to polyuria and polydipsia, and possibly dehydration. With the
excessive loss of free water, sodium levels rise above the expected reference range of 136 to 145
mEq/L.
459. A nurse is planning care to address nutritional needs for a preschooler who has cystic
fibrosis. Which of the following interventions should the nurse include in the plan?
Answer: Increase fat content in the childs diet to 40% of total calories.: A child who has cystic
fibrosis is unable to properly digest fats due to fibrosis of the pancreas and limited secretion of
pancreatic enzymes. The nurse should increase the child's fat intake to 35% to 40% of total
caloric intake.
460. A nurse is caring for a toddler who has acute otitis media and a temperature of 40 C (104 F).
After administering acetaminophen, which of the following actions should the nurse plan to take
to reduce the toddler's temperature?
Answer: Dress the toddler in minimal clothing: The nurse should recognize that dressing the
toddler in minimal clothing will expose the skin to air and maximize heat evaporation from the
skin, thus reducing the toddler's temperature.

461. A nurse is teaching a school-age child and their parent about postoperative care following
cardiac catheterization. Which of the following instructions should the nurse include?
Answer: "Wait 3 days before taking a tub bath.": The child should keep the site clean and dry for
at least 3 days to reduce the risk of infection. Tub baths should be avoided for 3 days to avoid
immersion of the incision in water.
462. A nurse is assessing an infant who has pneumonia. Which of the following findings is the
priority for the nurse to report to the provider?
Answer: Nasal flaring: When using the airway, breathing, and circulation approach to client
care, the nurse should determine that the priority finding to report to the provider is nasal flaring.
Nasal flaring indicates the infant is experiencing acute respiratory distress.
463. A nurse is teaching the parents of a toddler who has cognitive impairment about toilet
training. Which of the following instructions should the nurse include in the teaching?
Answer: "Award your child with a sticker when they sit on the potty chair.": A child who has a
cognitive impairment learns through shaping behaviors. The parents should reward the child for
sitting on the potty chair as a reinforcement of the desired behavior of continence. As the child
repeats this action, the parents can gradually decrease this reward and then give rewards for the
next step in the task, such as voiding while sitting on the potty chair.
464. EDKH
Answer: Epstein Didn't Kill Himself

Document Details

  • Subject: Nursing
  • Exam Authority: ATI
  • Semester/Year: 2019

Related Documents

The module download was not found!

person
Emma Thompson View profile
Close

Send listing report

highlight_off

You already reported this listing

The report is private and won't be shared with the owner

rotate_right
Close
rotate_right
Close

Send Message

image
Close

My favorites

image
Close

Application Form

image
Notifications visibility rotate_right Clear all Close close
image
image
arrow_left
arrow_right